AAFP questions

अब Quizwiz के साथ अपने होमवर्क और परीक्षाओं को एस करें!

A 39-year-old female presents with lower abdominal/pelvic pain. On examination, with the patient in a supine position, you palpate the tender area of her lower abdomen. When you have her raise both legs off the table while you palpate the abdomen, her pain intensifies. Which one of the following is the most likely diagnosis? A. Appendicitis B. A hematoma within the abdominal wall musculature C. Diverticulitis D. Pelvic inflammatory disease E. An ovarian cyst

A reduction of the pain caused by abdominal palpation when the abdominal muscles are tightened is known as Carnett's sign. If the cause of the pain is visceral, the taut abdominal muscles may protect the locus of pain. In contrast, intensification of pain with this maneuver points to a source of pain within the abdominal wall itself.

healthy 68-year-old male is seen in December for a routine examination. A review of his immunizations indicates that he received a standard dose of inactivated influenza vaccine at the health clinic in September. He received 23-valent pneumococcal vaccine (Pneumovax 23) at age 65. He should now receive which one of the following? A. High-dose influenza vaccine B. 13-valent pneumococcal conjugate vaccine (Prevnar 13) C. 23-valent pneumococcal vaccine D. No vaccines at this time

B. 13-valent pneumococcal conjugate vaccine (Prevnar 13) The Advisory Committee on Immunization Practices advises that the 13-valent pneumococcal vaccine be given in addition to the 23-valent vaccine, preferably before the 23-valent vaccine. Only one dose of influenza vaccine is recommended per season. A single dose of 23-valent pneumococcal vaccine is all that is required.

Which one of the following is true regarding NSAIDs? A. They are cardioprotective B. They should be avoided in persons with cirrhotic liver disease C. They are not safe in pregnancy D. They are not safe in lactating women

B. NSAIDs are prescribed commonly and many are available over the counter. It is important for clinicians to understand when they are not appropriate for clinical use. They should be avoided, if possible, in persons with hepatic cirrhosis (SOR C). While hepatotoxicity with NSAIDs is rare, they can increase the risk of bleeding in cirrhotic patients, as they further impair platelet function. In addition, NSAIDs decrease blood flow to the kidneys and can increase the risk of renal failure in patients with cirrhosis. NSAIDs differ from aspirin in terms of their cardiovascular effects. They have the potential to increase cardiovascular morbidity, worsen heart failure, increase blood pressure, and increase events such as ischemia and acute myocardial infarction. There are no known teratogenic effects of NSAIDs in humans. This drug class is considered to be safe in pregnancy in low, intermittent doses, although discontinuation of NSAID use within 6-8 weeks of term is recommended. Ibuprofen, indomethacin, and naproxen are considered safe for lactating women, according to the American Academy of Pediatrics.

A 65-year-old female is admitted to the hospital for a carotid endarterectomy and you are asked to make preoperative recommendations in advance of her surgery scheduled for tomorrow. She takes only low-dose aspirin. The physical examination is normal, including her blood pressure, as is an EKG. She has good exercise capacity and denies any symptoms of angina. You judge her to be stable for surgery. Which one of the following should you recommend that the patient start today? A. An ACE inhibitor B. A β-blocker C. A statin D. A diuretic

C If recommended prior to surgery, β-blockers should be started several weeks beforehand and carefully titrated. They may be harmful if initiated in the immediate perioperative period. Statins are recommended in the perioperative period for vascular surgery regardless of other cardiac risk factors; a statin would ideally have been initiated previously in this case, but may still be started in the immediate preprocedural period. There is no specific indication in this case for an ACE inhibitor.

One week after returning from a Caribbean vacation, a 43-year-old female presents to a walk-in clinic with a complaint of redness and itching on the sole of her foot. She recalls experiencing a stinging sensation in the same area while she was wading in the surf on the day before she was to return home, but was unable to see any sign of injury immediately following the incident. Since her return the itching has intensified and the red area has enlarged. The most likely cause of this condition is a: A. filarial nematode B. jellyfish C. hookworm D. roundworm E. tapeworm

C. hookworm When third-stage hookworm larvae, most commonly of the species infecting dogs and cats, penetrate the skin and migrate through the dermis, they create the serpiginous, erythematous tracks characteristic of cutaneous larva migrans. Although this dermatosis can occur in northern areas when conditions are ideal, it is most often encountered in tropical and semitropical regions such as the Caribbean, Africa, Asia, and South America. Travelers to beach environments where pet feces have been previously deposited are most at risk because of the direct contact of bare skin with the sand. As in this case, a stinging or itching sensation may be noted upon penetration; this is followed by the development of the creeping eruption, which usually appears 1-5 days later, although the onset may be delayed for up to a month. The larvae will not develop in the human host, so the infection is self-limited, usually resolving within weeks to months. Treatment with antihelminthic drugs can greatly reduce the clinical course. Preventive measures include treatment of infected dogs and cats and limiting exposure to contaminated soil by wearing shoes and protective clothing.

Sympathomimetic decongestants such as pseudoephedrine and phenylephrine can be problematic in elderly patients because they can A. decrease blood pressure B. cause bradycardia C. worsen existing urinary obstruction D. enhance the anticholinergic effects of other medications E. enhance the sedative effects of other medications

C. worsen existing urinary obstruction Sympathomimetic agents can elevate blood pressure and intraocular pressure, may worsen existing urinary obstruction, and adversely interact with β-blockers, methyldopa, tricyclic antidepressants, oral hypoglycemic agents, and MAOIs. They also speed up the heart rate. First-generation nonprescription antihistamines can enhance the anticholinergic and sedative effects of other medications.

A 4-week-old white male is brought to your office with a 2-week history of increasing dyspnea, cough, and poor feeding. The child appears nontoxic and is afebrile. On examination you note conjunctivitis, and a chest examination reveals tachypnea and crackles. A chest film shows hyperinflation and diffuse interstitial infiltrates and a WBC count reveals eosinophilia.

Chlamydial pneumonia is usually seen in infants 3-16 weeks of age, and these patients frequently have been sick for several weeks. The infant appears nontoxic and is afebrile, but is tachypneic with a prominent cough. The physical examination will reveal diffuse crackles with few wheezes, and conjunctivitis is present in about 50% of cases. A chest film will show hyperinflation and diffuse interstitial or patchy infiltrates. Staphylococcal pneumonia has a sudden onset. The infant appears very ill and has a fever, and initiallymay have an expiratory wheeze simulating bronchiolitis. Signs of abdominal distress, tachypnea, dyspnea, and localized or diffuse bronchopneumonia or lobar disease may be present. The WBC count will show a prominent leukocytosis. Respiratory syncytial virus infections start with rhinorrhea and pharyngitis, followed in 1-3 days by acough and wheezing. Auscultation of the lungs will reveal diffuse rhonchi, fine crackles, and wheezes, but the chest film is often normal. If the illness progresses, coughing and wheezing increase, air hunger and intercostal retractions develop, and evidence of hyperexpansion of the chest is seen. In some infants the course of the illness may be similar to that of pneumonia. Rash or conjunctivitis may occur occasionally, and fever is an inconsistent sign. The WBC count will be normal or elevated, and the differential may be normal or shifted either to the right or left. Chlamydial infections can be differentiated from respiratory syncytial virus infections by a history of conjunctivitis, the subacute onset and absence of fever, and the mild wheezing. There may also be eosinophilia. Parainfluenza virus infection presents with typical cold symptoms. Eight percent of infections affect the upper respiratory tract. In children hospitalized for severe respiratory illness, parainfluenza viruses account for about 50% of the cases of laryngotracheitis and about 15% each of the cases of bronchitis, bronchiolitis, and pneumonia.

A 42-year-old male with a 4-year history of multiple sclerosis (MS) presents with an acute attack manifested by ataxia, incoordination, and dysarthria. Which one of the following is indicated for managing this flare-up of his MS? A. Fingolimod (Gilenya) B. Glatiramer (Copaxone) C. Interferon-β (Avonex, Betaseron) D. Methylprednisolone (Medrol) E. Pramipexole (Mirapex)

Corticosteroids, either orally or parenterally, are the first-line treatment for acute exacerbations of multiple sclerosis (MS) (SOR A). A Cochrane review found no significant differences in outcomes based on the route of administration. Disease-modifying agents such as interferon beta, glatiramer, and immunosuppressants such as fingolimod may decrease the frequency of exacerbations and slow the progression of MS but are not the agents of first choice for treatment of acute flareups. Pramipexole does not have a primary role in the treatment of MS, although it might be used to treat certain specific symptoms as an adjunct therapy.

A 53-year-old female without risk factors for colorectal cancer undergoes a screening colonoscopy. A high-quality examination reveals five 3- to 7-mm sessile polyps in the sigmoid and rectal areas. Biopsy results show that they are hyperplastic polyps. No other abnormalities are noted. When should this patient have her next colonoscopy? A. 1 year B. 3 years C. 5 years D. 10 years E. No further colonoscopies needed

D. 10 years Hyperplastic polyps <10 mm in size in the rectum and sigmoid colon carry a low risk for developing into colon cancer. If they are the only finding, colonoscopy may be repeated in 10 years.

You are caring for a 60-year-old female with Crohn's disease that is well controlled by infliximab (Remicade). As your staff updates her immunization status, which one of the following should be kept in mind? (check one) A. Hepatitis A vaccine is contraindicated B. Pneumococcal vaccine is contraindicated C. Tetanus toxoid is contraindicated D. Zoster vaccine is contraindicated

D. Zoster vaccine is contraindicated Zoster vaccine, a live attenuated virus vaccine, is contraindicated in this patient due to her immunocompromised state. The other vaccines listed are safe and particularly recommended for patients with inflammatory bowel disease, given their increased susceptibility to infections. Immunosuppression is defined as: • Treatment with glucocorticoids (treatment with the equivalent of 20 mg/day of prednisone for 2 weeks or more, and discontinuation within the previous 3 months) • Ongoing treatment with effective doses of 6-MP/azathioprine or discontinuation within the previous 3 months • Treatment with methotrexate or discontinuation within the previous 3 months • Treatment with infliximab or discontinuation within the previous 3 months • Significant protein-calorie malnutrition

A 21-year-old primigravida at 28 weeks gestation complains of the recent onset of itching. On examination she has no obvious rash. The pruritus started on her palms and soles and spread to the rest of her body. Laboratory evaluation reveals elevated serum bile acids and mildly elevated bilirubin and liver enzymes. The most effective treatment for this condition is: A. triamcinolone (Kenalog) cream B. cholestyramine (Questran) C. diphenhydramine (Benadryl) D. doxylamine succinate E. ursodiol (Actigall)

E. ursodiol (Actigall) This patient's symptoms and laboratory values are most consistent with intrahepatic cholestasis of pregnancy. Ursodiol has been shown to be highly effective in controlling the pruritus and decreased liver function (SOR A) and is safe for mother and fetus. Topical antipruritics and oral antihistamines are not very effective. Cholestyramine may be effective in mild or moderate intrahepatic cholestasis, but is less effective and safe than ursodiol.

A 70-year-old male complains of lower-extremity pain. Increased pain with which one of the following would be most consistent with lumbar spinal stenosis? A. Lumbar spine extension B. Lumbar spine flexion C. Internal hip rotation D. Pressure against the lateral hip and trochanter E. Walking uphill

Extension that increases lumbar lordosis decreases the cross-sectional area of the spinal canal, thereby compressing the spinal cord further. Walking downhill can cause this. Spinal flexion that decreases lordosis has the opposite effect, and will usually improve the pain, as will sitting. Pain with internal hip rotation is characteristic of hip arthritis and is often felt in the groin. Pain in the lateral hip is more typical of trochanteric bursitis. Increased pain walking uphill is more typical of vascular claudication.

A 12-month-old male is brought to your office for a routine well child visit. His father has epilepsy and takes seizure medication. Which one of the following vaccines will slightly increase the child's risk of a febrile seizure for up to 2 weeks after administration?

Fever and febrile seizures may occur after administration of several vaccines. Postimmunization seizures, especially febrile seizures, occur at a higher rate in children who have a past history of seizures or a first-degree relative with a history of seizures. The benefits of the vaccines outweigh the risks, so they are not contraindicated in this situation, although the parents need to be cautioned about the increased risk of seizure. Of the vaccines listed, the only one likely to put the child at risk for a seizure up to 2 weeks after administration is the MMR vaccine. Specifically, it is the measles component of the vaccine that is the potential culprit. A temperature of 39.4°C (103°F) or higher develops in approximately 5%-15% of susceptible vaccine recipients, usually 6-12 days after receipt of MMR vaccine. The fever generally lasts 1-2 days but may last up to 5 days.

A 49-year-old male brings you a copy of his laboratory results obtained during an insurance examination. The patient says he feels fine, but his bilirubin level was 2.5 mg/dL (N <1.0). He says he averages 5 alcoholic beverages per week and takes no medications other than occasional ibuprofen. On examination he is not jaundiced and has no scleral icterus, and the remainder of the examination is within normal limits, including palpation of the liver and spleen. Laboratory testing reveals a normal CBC, normal liver enzyme levels, and normal serum haptoglobin. Bilirubin fractionation reveals an indirect level of 2.0 mg/dL and a direct level of 0.5 mg/dL (N<0.4). The most likely diagnosis is

Gilbert's syndrome is a hereditary condition associated with unconjugated hyperbilirubinemia (usually with a bilirubin level <5.0 mg/dL). The bilirubin level increases with infection, exertion, and fasting. Patients are asymptomatic and have otherwise normal liver function studies. The differential diagnosis includes hemolytic anemias, which cause a decrease in serum haptoglobin, an increase in lactate dehydrogenase, and/or CBC abnormalities, particularly on the peripheral smear.

A 43-year-old female complains of easy bruising. She is otherwise asymptomatic. A CBC reveals a platelet count of 23,000/mm3 (N 150,000-450,000). A peripheral smear reveals giant platelets. A workup is negative for autoimmune causes, including Graves disease, HIV, Epstein-Barr virus, cytomegalovirus, varicella zoster, hepatitis C, and Helicobacter pylori. She is on no prescription or over-the-counter medications and denies alcohol or drug use. Which one of the following would be the most appropriate initial management? A. Platelet transfusion B. Corticosteroids C. Thrombopoietin-receptor agonists D. A bone marrow biopsy E. Splenectomy

Immune (idiopathic) thrombocytopenic purpura is an acquired immune-mediated disorder defined as isolated thrombocytopenia not found to have another cause. Treatment is usually restricted to severe thrombocytopenic cases (platelet count <50,000/mm3) unless there is evidence of acute bleeding. Corticosteroids are considered the first-line therapy. Intravenous immunoglobulin and rituximab have also been used as first-line agents. Second-line therapies include thrombopoietin-receptor agonists and splenectomy. Further evaluation, including a bone marrow biopsy, to rule out myelodysplastic syndrome and lymphoproliferative disorders is indicated in patients over the age of 60. Platelet transfusion is not indicated in the absence of hemorrhage or a need for surgery.

Medicare pays for which one of the following? A. Routine dental care B. Custodial nursing-home care C. Hearing aids D. Screening mammography

Medicare pays for some preventive measures, including pneumococcal vaccine, influenza vaccine, annual mammography, and a Papanicolaou test every 3 years. Medicare does not pay for custodial care, nursing-home care (except limited skilled nursing care), dentures, routine dental care, eyeglasses, hearing aids, routine physical checkups and related tests, or prescription drugs.

A 19-year-old college wrestler presents with cellulitis of his left arm extending from a small pustule on his hand to the axilla. He appears acutely ill and has a temperature of 38.9°C (102.0°F). His WBC count is 22,000/mm3 (N 4300-10,800). He is admitted to the hospital. The initial drug of choice for this patient would be A. ciprofloxacin (Cipro) B. clindamycin (Cleocin) C. doxycycline D. trimethoprim/sulfamethoxazole E. vancomycin

Methicillin-resistant Staphylococcus aureus (MRSA) is the predominant cause of suppurative skin and soft-tissue infection. While community-acquired strains have been susceptible to many antibiotics, clindamycin is associated with Clostridium difficile enterocolitis, trimethoprim/sulfamethoxazole is usually used orally only for outpatient treatment, and doxycycline and minocycline are often effective clinically but seldom used for serious infections. Resistance to quinolones is increasing and may emerge during treatment. Vancomycin given parenterally is generally still the drug of choice for hospitalized patients.

Mild cognitive impairment is characterized by which one of the following? (check one) A. Localized motor dysfunction B. Impairment in at least one activity of daily living C. Impairment in at least one instrumental ADL D. The presence of the APO E4 allele E. Objective evidence of memory decline

Mild cognitive impairment is an intermediate stage between normal cognitive function and dementia. Motor function remains normal. The presence of the APO E4 allele is a risk factor, but is not necessary for a diagnosis. Patients have essentially normal functional activities but there is objective evidence of memory impairment, and the patient may express concerns about cognitive decline.

Which one of the following comorbid conditions increases the risk that latent tuberculosis infection will progress to active disease? A. Hypertension B. Lung cancer C. Obesity D. Coronary artery disease E. Hyperlipidemia

Risk factors for progression from latent to active tuberculosis include lung cancer, diabetes mellitus, alcoholism, recent contact with a person who has an active tuberculosis infection, any condition treated with immunosuppressive therapy, and lung parenchymal diseases such as COPD, silicosis, or lung cancer. The medically underserved and those in low-income groups are also more at risk of progression, as well as children under age 5 and individuals weighing less than 90% of their ideal minimum body weight.

2 y.o. not using her left arm and holding it slightly pronated, flexed, and close to her body.Physical examination of the child's clavicle, shoulder, wrist, and hand do not elicit any signs of pain or change in function. She does seem to have some tenderness near the lateral elbow and resists your attempts to examine that area. There is no ecchymosis, swelling, or deformity of the elbow. What do you do next?

Radial head subluxation, or nursemaid's elbow, is the most common orthopedic condition of the elbow in children 1-4 years of age, although it can be encountered before 1 year of age and in children as old as 9 years of age. The mechanism of injury is partial displacement of the radial head when the child's arm undergoes axial traction while in a pronated and fully extended position. The classic history includes a caregiver picking up (or pulling) a toddler by the arm. In half of all cases, however, no inciting event is recalled. As long as there are no outward signs of fracture or abuse it is considered safe and appropriate to attempt reduction of the radial head before moving on to imaging studies. With the child's elbow in 90° of flexion, the hand is fully supinated by the examiner and the elbow is then brought into full flexion. Usually the child will begin to use the affected arm again within a couple of minutes. If ecchymosis, significant swelling, or pain away from the joint is present, or if symptoms do not improve after attempts at reduction, then a plain radiograph is recommended.

A healthy 18-year-old female sees you for a preparticipation evaluation and well care visit prior to soccer season. She has no significant previous medical history and no current problems. She says she is not sexually active. She has completed the HPV vaccine series. Which one of the following would be most appropriate for cervical cancer screening for this patient?

The U.S. Preventive Services Task Force recommends against screening for cervical cancer for women younger than 21, for women over the age of 65 who have had adequate screening in the recent past and are not at high risk, and for women who have had a hysterectomy with removal of the cervix and no history of CIN 2 or 3 or cervical cancer (USPSTF D recommendation). Women between the ages of 21 and 65 can be screened every 3 years with cytology alone, or the interval can be increased to 5 years after age 30 by using a combination of cytology and HPV testing (USPSTF A recommendation). The history of HPV vaccination is not a factor in screening decisions. Other organizations such as the American Cancer Society and the American College of Obstetricians and Gynecologists have similar guidelines.

A 60-year-old female has a strong family history of breast cancer and is considering tamoxifen (Soltamox) to reduce her risk. Which one of the following is an effect associated with this treatment that should be included in the shared decision-making discussion with the patient?

The U.S. Preventive Services Task Force recommends that the selective estrogen receptor modulators tamoxifen and raloxifene be offered to women at high risk for breast cancer and low risk for adverse medication effects (B recommendation). This reduces the incidence of invasive breast cancer by 7-9 events per 1000 women over 5 years. Tamoxifen has been shown to be more beneficial than raloxifene. Potential harms include an increase of 4-7 events of venous thromboembolism per 1000 women over 5 years. Tamoxifen increases the risk more than raloxifene. Tamoxifen also reduces bone fractures but increases the incidence of endometrial cancer, leg cramps, bladder control issues, vasomotor symptoms, and vaginal dryness, itching, and discharge.

An asymptomatic 60-year-old male sees you for a health maintenance visit. His past medical history is significant for hypertension and hyperlipidemia. His medications include chlorthalidone, 25 mg daily, and atorvastatin (Lipitor), 20 mg daily. He smoked 2 packs of cigarettes a day for 20 years but quit 5 years ago. The physical examination is normal. Laboratory findings include a normal basic metabolic panel, a cholesterol level of 210 mg/dL, an HDL-cholesterol level of 34 mg/dL, an LDL-cholesterol level of 150 mg/dL, and a triglyceride level of 200 mg/dL. Which one of the following screening tests is recommended by the U.S. Preventive Services Task Force for this patient? A. Prostate-specific antigen (PSA) B. A bone density test C. Abdominal ultrasonography D. Low-dose chest CT E. Carotid ultrasonography

The U.S. Preventive Services Task Force (USPSTF) recommends screening smokers for lung cancer with low-dose CT. Patients should be age 55-80 and healthy. They should be current smokers or have quit within the past 15 years, and have a 30-pack-year history of smoking. The screening test is low-dose CT of the chest. Abdominal ultrasonography to screen for abdominal aneurysms is recommended for any male age 65-75 who has ever smoked (USPSTF B recommendation). A bone density test screens for osteoporosis and is recommended for women age 65 or older or in younger women at increased risk. The USPSTF recommends against PSA testing (D recommendation) for prostate cancer, as well as screening for carotid artery stenosis.

A 24-year-old female presents to your clinic with a 5-day history of fever to 103°F. She has no localizing symptoms or overt physical findings. Initial testing shows an elevated WBC count with a disproportionate number of reactive lymphocytes. Which one of the following conditions is the most likely cause of these findings? A. Bacterial infection B. Connective tissue disease C. Lymphoma D. Viral infection

The conditions that result in an absolute increase in lymphocytes are divided into primary causes (usually neoplastic hyperproliferation) and secondary or reactive causes. The presence of reactive lymphocytes will often be reported on a manual differential, since they have a distinctive appearance. The most common conditions that produce a reactive lymphocytosis are viral infections. Most notable are Epstein-Barr virus, infectious mononucleosis, and cytomegalovirus. Other viral infections known to cause this finding include herpes simplex, herpes zoster, HIV, hepatitis, and adenovirus. Connective tissue disease can infrequently cause a reactive lymphocytosis, but other signs or symptoms are usually present. Bacterial infections more commonly result in an increase in neutrophils. One exception to this is Bordetella pertussis, which has been known to cause absolute lymphocyte counts of up to 70,000/μL. This infection is associated with classic symptoms that this patient does not have.

Which one of the following is a common cause of prerenal acute kidney injury? (ch A. Acute tubular necrosis B. Diuretic overuse C. Glomerulonephritis D. Neurogenic bladder E. Prostate hypertrophy

The diagnosis of acute kidney injury (AKI) is based on elevated serum creatinine levels and is often associated with a reduction in urine output (SOR C). The causes of AKI are commonly divided into three categories: prerenal, intrinsic renal, and postrenal (SOR C). Prerenal AKI is most commonly due to decreased renal perfusion, often because of volume depletion. In addition to vomiting and diarrhea, overuse of diuretics can lead to prerenal AKI. Intrinsic renal AKI is caused by a process within the kidneys. Glomerulonephritis and acute tubular necrosis are types of intrinsic AKI. Postrenal AKI refers to a process distal to the kidneys and is most often caused by inadequate drainage of urine. Neurogenic bladder and prostate hypertrophy contribute to extrarenal obstruction.

An 11-year-old female has been diagnosed with "functional abdominal pain" by a pediatric gastroenterologist. Her mother brings her to see you because of concerns that another diagnosis may have been overlooked despite a very thorough and completely normal evaluation for organic causes. Which one of the following would you recommend? A. A trial of inpatient hospital admission B. Increased testing and levels of referral until a true diagnosis is reached C. Removing the child from school and activities whenever symptoms occur D. Medications to eradicate symptoms E. Stress reduction and participation in usual activities as much as possible

The diagnosis of functional abdominal pain is made when no structural, infectious, inflammatory, or biochemical cause for the pain can be found. It is the most common cause of recurrent abdominal pain in children 4-16 years of age. The use of medications may be helpful in reducing (but rarely eradicating) functional symptoms, and remaining open to the possibility of a previously unrecognized organic disorder is appropriate. However, continuing to focus on organic causes, invasive tests, or physician visits can actually perpetuate a child's complaints and distress. It is estimated that approximately 30%-50% of children with functional abdominal pain will have resolution of their symptoms within 2 weeks of diagnosis. Recommendations for managing this problem include focusing on participation in normal age-appropriate activities, reducing stress and addressing emotional distress, and teaching the family to cope with the symptoms in a way that prevents secondary gain on the part of the child.

A 52-year-old healthy male presents with a 2½-week history of diarrhea, consisting of 4-6 watery stools daily. He is afebrile and his examination is normal. You recommend symptomatic care. Two days later the laboratory notifies you that Salmonella is growing in his stool culture. You call the patient and he remains free of fever but with ongoing diarrhea. Which one of the following would you recommend? A. Azithromycin (Zithromax) B. Ciprofloxacin (Cipro) C. Clindamycin (Cleocin) D. Doxycycline E. No treatment

The recommended management for patients who have non-severe Salmonella infection and are otherwise healthy is no treatment. Patients with high-risk conditions that predispose to bacteremia, and those with severe diarrhea, fever, and systemic toxicity or positive blood cultures should be treated with levofloxacin, 500 mg once daily for 7-10 days (or another fluoroquinolone in an equivalent dosage), or with a slow intravenous infusion of ceftriaxone, 1-2 g once daily for 7-10 days (14 days in patients with immunosuppression).

A factory worker sustains a forced flexion injury of the distal interphalangeal (DIP) joint, resulting in a small bone fragment at the dorsal surface of the proximal distal phalanx (mallet fracture). Which one of the following is the most appropriate management strategy?

The recommended treatment for a mallet fracture is splinting the distal interphalangeal (DIP) joint in extension (SOR B). The usual duration of splinting is 8 weeks. It is important that extension be maintained throughout the duration of treatment because flexion can affect healing and prolong the time needed for treatment. If the finger fracture involves >30% of the intra-articular surface, referral to a hand or orthopedic surgeon can be considered. However, conservative therapy appears to have outcomes similar to those of surgical treatment and therefore is generally preferred.

An 80-year-old female is seen for progressive weakness over the past 8 weeks. She says she now has difficulty with normal activities such as getting out of a chair and brushing her teeth. Her medical problems include hypertension, diabetes mellitus, and hyperlipidemia. Her medications include glipizide (Glucotrol), simvastatin (Zocor), and lisinopril (Prinivil, Zestril). Findings on examination are within normal limits except for diffuse proximal muscle weakness and normal deep tendon reflexes. A CBC, urinalysis, erythrocyte sedimentation rate, TSH level, and serum electrolyte levels are normal. Her blood glucose level is 155 mg/dL and her creatine kinase level is 1200 U/L (N 40-150).

This patient is most likely suffering from a drug-induced myopathy caused by simvastatin, which is associated with elevated creatine kinase. Polymyalgia rheumatica is usually associated with an elevated erythrocyte sedimentation rate. Guillain-Barré syndrome is associated with depressed deep tendon reflexes. This case has no clinical features or laboratory findings that suggest ketoacidosis.

While performing a routine physical examination on a 42-year-old female you discover an apparent nodule in the left lobe of the thyroid measuring approximately 1 cm in diameter, which is confirmed on ultrasonography. The most appropriate next step in the evaluation of this finding is a

Thyroid nodules >1 cm that are discovered incidentally on examination or imaging studies merit further evaluation. Nodules <1 cm should also be fully evaluated when found in patients with a family history of thyroid cancer, a personal history of head and neck irradiation, or a finding of cervical node enlargement. Reasonable first steps include measurement of TSH or ultrasound examination. The American Thyroid Association's guidelines recommend that TSH be the initial evaluation (SOR A) and that this be followed by a radionuclide thyroid scan if results are abnormal. Diagnostic ultrasonography is recommended for all patients with a suspected thyroid nodule, a nodular goiter, or a nodule found incidentally on another imaging study (SOR A). Routine measurement of serum thyroglobulin or calcitonin levels is not currently recommended.

A 42-year-old female presents to the emergency department with a 2-hour history of palpitations. Her physical examination is normal except for what seems to be a regular rhythm tachycardia and a blood pressure of 84/54 mm Hg. An EKG reveals a regular narrow-complex tachycardia at a rate of 180 beats/min without clear atrial activity. The optimal treatment for this patient is (check one) A. intravenous adenosine (Adenocard) B. intravenous amiodarone (Cordarone) C. intravenous diltiazem D. intravenous verapamil E. electrical cardioversion

Vagal maneuvers and administration of adenosine are useful in the diagnosis and treatment of narrow-complex supraventricular tachycardias. Adenosine, a very short-acting endogenous nucleotide that blocks atrioventricular nodal conduction, terminates nearly all atrioventricular nodal reentrant tachycardias and atrioventricular reciprocating tachycardias, as well as up to 80% of atrial tachycardias. Although intravenous verapamil and diltiazem, which also block the atrioventricular node, have a potential diagnostic and therapeutic use in narrow-complex tachycardia, they may cause hypotension and thus are not a first choice in the emergency setting. Electrical cardioversion is reserved for patients who do not respond to adenosine. Antiarrhythmic agents are rarely necessary in the early management of supraventricular tachycardias, with the exception of the management of arrhythmias that have caused hemodynamic instability and that have not responded to electrical cardioversion. In these cases, procainamide and ibutilide can be used.

In older patients with aortic stenosis and a systolic murmur, which one of the following would be most concerning? A. Weight loss B. Frequent urination C. Jaundice D. Worsening headache E. Exertional dyspnea

When symptoms begin to appear in a patient with aortic stenosis the prognosis worsens. It is therefore important to be aware of systolic murmurs in older patients presenting with exertional dyspnea, chest pain, or dizziness. This can be the first presentation of a downward spiral and the need for rapid valve replacement. Weight loss, frequent urination, jaundice, and worsening headache are not as closely associated with a generally worse outlook for patients with aortic stenosis.

In addition to exercise, which one of the following vitamin supplements is recommended by the U.S. Preventive Services Task Force to help prevent falls in elderly patients living at home?

Vitamin D supplementation helps prevent falls in community-dwelling adults age 65 and older, although the mechanism is not clearly understood. Supplementation is recommended by the U.S. Preventive Services Task Force

A 24-year-old female presents to the emergency department because she thinks she is having an allergic reaction to her medication for depression. About 3 hours after taking her first dose of citalopram (Celexa) she noted extreme anxiety, agitation, palpitations, and a dry mouth. On examination she has a blood pressure of 180/110 mm Hg, a pulse rate of 120 beats/min, a respiratory rate of 24/min, and a temperature of 37.2°C (99.0°F). Her pupils are dilated and she has slow, continuous horizontal eye movements. Marked hyperreflexia is noted in the lower extremities. In addition to supportive care, the patient should be given intravenous...

diazepam Serotonin syndrome is a result of increased serotonergic activity in the central nervous system and may be life-threatening. It is usually a combination of autonomic hyperactivity, neuromuscular abnormality, and mental status changes. The most common group of medications that may cause this is the SSRIs. Serotonin syndrome most commonly occurs in the first 24 hours of treatment. Patients often present with agitation and confusion, tachycardia, and elevated blood pressure, as well as a dry mouth. While there are usually no focal neurologic findings, hyperreflexia and even spontaneous clonus may be seen. The finding of slow, horizontal movement of the eyes is also helpful in making the diagnosis. The initial management is to discontinue the offending agent, begin supportive care, and attempt to calm the patient verbally. Many times medication is needed, and the drug of choice is an intravenous benzodiazepine such as lorazepam or diazepam. If treatment for tachycardia or hypertension is needed, propranolol should not be used due to its longer activity. Haloperidol should be avoided, as it may actually increase anticholinergic activity. Flumazenil is rarely used, although it has been used for tricyclic antidepressant overdosage, and it carries a significant risk of inducing seizures. If the patient does not respond to calming with benzodiazepines, the antidote would be cyproheptadine.

A U.S. Preventive Services Task Force "D" recommendation indicates A. high certainty that the net benefit is substantial B. high certainty that the net benefit is moderate C. moderate or high certainty that the service has no net benefit or that the harms outweigh the benefits D. that the decision to provide the service should be based on professional judgment and patient preferences E. that current evidence is insufficient to assess the balance of benefits and harms of the service

A "D" recommendation means the U.S. Preventive Services Task Force (USPSTF) recommends against the service. There is moderate or high certainty that the service has no net benefit or that the harms outweigh the benefits. An "I" recommendation means the USPSTF concludes that the evidence is lacking, of poor quality, or conflicting, and the balance of benefits and harms cannot be determined. A "C" recommendation means the USPSTF recommends selectively offering or providing this service to individual patients based on professional judgment and patient preferences. There is at least moderate certainty that the net benefit is small. A "B" recommendation means the USPSTF recommends the service. There is high certainty that the net benefit is moderate or there is moderate certainty that the net benefit is moderate to substantial. An "A" recommendation means the USPSTF recommends the service and there is high certainty that the net benefit is substantial. The highest levels of evidence and most recent evidence available are used by the USPSTF in making all of its recommendations.

You have prescribed oral iron replacement for a 46-year-old female with iron deficiency anemia related to heavy menses. She wants to be sure that the iron she takes will be absorbed well. Which one of the following would you suggest for improving iron absorption? A. Calcium B. Vitamin C C. Coffee D. Tea

B. Vitamin C Taking oral iron with vitamin C or a meal high in meat protein increases iron absorption. Calcium and coffee both decrease iron absorption, but not as much as tea, which can reduce absorption of oral iron by as much as 90%.

A 7-year-old male presents with a fever of 38.5°C (101.3°F), a sore throat, tonsillar inflammation, and tender anterior cervical adenopathy. He does not have a cough or a runny nose. His younger sister was treated for streptococcal pharyngitis last week and his mother would like him to be treated for streptococcal infection. Which one of the following is true concerning this situation? A. Empiric antibiotic treatment for streptococcal pharyngitis is warranted. B. The chance of this patient having a positive rapid antigen detection test for Streptococcus is <50%. C. There is a generalized consensus among the various national guidelines for management of pharyngitis. D. The patient should have a tonsillectomy when he recovers from this infection. E. The family dog should be treated for streptococcal infection.

A. The patient has a score of 5 under the Modified Centor scoring system for management of sore throat. Patients with a score ≥4 are at highest risk (at least 50%) of having group A β-hemolytic streptococcal (GABHS) pharyngitis, and empiric treatment with antibiotics is warranted. Various national and international organizations disagree about the best way to manage pharyngitis, with no consensus as to when or how to test for GABHS and who should receive treatment. The minimal benefit seen with tonsillectomy in reducing the incidence of recurrent GABHS pharyngitis does not justify the risks or cost of surgery. Treatment of pets for the prevention of GABHS infection has proven ineffective.

A 55-year-old white male comes to your office with weakness and a headache. He also describes an annoying pruritus that occurs frequently after he takes a hot shower. The physical examination is remarkable for the presence of an enlarged spleen. He has a hemoglobin level of 21 g/dL (N 12-16) and a hematocrit of 63% (N 36-48). To confirm your clinical diagnosis, you obtain additional studies. Which one of the following would be most consistent with the most likely diagnosis in this patient? A. A low serum erythropoietin level B. A low platelet count C. A low arterial oxygen concentration D. An elevated carboxyhemoglobin level

A. A low serum erythropoietin level The patient described in this case has polycythemia vera. Pruritus after a hot shower (aquagenic pruritus) and the presence of splenomegaly helps to clinically distinguish polycythemia vera from other causes of erythrocytosis (hematocrit >55%). Specific criteria for the diagnosis of polycythemia vera include an elevated red cell mass, a normal arterial oxygen saturation (>92%), and the presence of splenomegaly. In addition, patients usually exhibit thrombocytosis (platelet count >400,000/mm3 ), leukocytosis (WBC>12,000/mm3 ), a low serum erythropoietin level, and an elevated leukocyte alkaline phosphatase score. High carboxyhemoglobin levels are associated with secondary polycythemia.

When compared to a figure-of-eight dressing, which one of the following modalities of treatment has been shown to have similar fracture-healing outcomes and increased patient satisfaction for nondisplaced mid-shaft clavicular fractures? A. A shoulder sling B. A short arm cast C. A long arm cast D. Operative fixation

A. A shoulder sling When compared to a figure-of-eight dressing, a sling has been shown to have similar fracture healing rates in patients with a nondisplaced midshaft clavicular fracture. In addition, a figure-of-eight dressing is uncomfortable and difficult to adjust, and patients have reported increased satisfaction when treated with a sling. Long and short arm casts are not appropriate options to manage a patient with a clavicular fracture. Operative treatment is an option to treat displaced midshaft fractures

A 30-year-old female is being evaluated for chronic pain, fatigue, muscle aches, and sleep disturbance. Which one of the following would be best for making a diagnosis of fibromyalgia? A. A structured symptom history B. Examination for tender points C. Laboratory testing D. A muscle biopsy E. Electromyography

A. A structured symptom history The American College of Rheumatology has defined diagnostic criteria for fibromyalgia based on the patient's symptoms. Previously, tender points on examination were the diagnostic criterion. Laboratory testing, muscle biopsies, and electromyography can be used to rule out other conditions.

A 3-week-old infant is brought to your office with a fever. He has a rectal temperature of 38.3°C (101.0°F), but does not appear toxic. The remainder of the examination is within normal limits. Which one of the following would be the most appropriate management for this patient? (check one) A. Admit to the hospital and obtain urine, blood, and CSF cultures, then start intravenous antibiotics B. Admit to the hospital and treat for herpes simplex virus infection C. Follow up in the office in 24 hours and admit to the hospital if not improved D. Order a CBC and a urinalysis with culture, and send the patient home if the results are normal

A. Admit to the hospital and obtain urine, blood, and CSF cultures, then start intravenous antibiotics Any child younger than 29 days old with a fever and any child who appears toxic, regardless of age, should undergo a complete sepsis workup and be admitted to the hospital for observation until culture results are known or the source of the fever is found and treated (SOR C). Observation only, with close follow-up, is recommended for nontoxic infants 3-36 months of age with a temperature <39.0°C (102.2°F) (SOR C). Children 29-90 days old who appear to be nontoxic and have negative screening laboratory studies, including a CBC and urinalysis, can be sent home with precautions and with follow-up in 24 hours (SOR B). Testing for neonatal herpes simplex virus infection should be considered in patients with risk factors, including maternal infection at the time of delivery, use of fetal scalp electrodes, vaginal delivery, cerebrospinal fluid pleocytosis, or herpetic lesions. Testing also should be considered when a child does not respond to antibiotics

A 40-year-old male is admitted to the hospital with a generalized rash consistent with Stevens-Johnson syndrome. His previous medical problems include obesity, gout, hypertension, type 2 diabetes mellitus, and depression. His medications include lisinopril (Prinivil, Zestril), allopurinol (Zyloprim), colchicine (Colcrys), metoprolol succinate (Toprol-XL), metformin (Glucophage), and venlafaxine (Effexor XR). Which one of these medications is most likely to be the cause of his Stevens-Johnson syndrome? A. Allopurinol B. Colchicine C. Lisinopril D. Metformin E. Venlafaxine

A. Allopurinol Stevens-Johnson syndrome is a rare, potentially life-threatening condition. There are many potential causes, including infection, vaccines, systemic disease, physical agents, food, and drugs. Allopurinol is the only drug listed that is a potential causative agent. Other drugs commonly cited as causes include antibiotics, antiepileptics, and NSAIDs. Of these, antibiotics are the most common alleged cause of Stevens-Johnson syndrome.

A 3-year-old female is brought to your office with coughing and a tactile fever. Her only other symptom is mild rhinorrhea. She has a temperature of 38.2°C (100.8°F) and is mildly tachypneic. Her vital signs are otherwise normal and she appears to be well and in no respiratory distress. Her examination is unremarkable except for decreased breath sounds and crackles in the right lower lung field. She has no allergies to medications. Which one of the following would be the most appropriate treatment? A. Amoxicillin B. Azithromycin (Zithromax) C. Cefdinir D. Moxifloxacin (Avelox) E. Ceftriaxone (Rocephin)

A. Amoxicillin Amoxicillin is the recommended first-line treatment for previously healthy infants and school-age children with mild to moderate community-acquired pneumonia (CAP) (strong recommendation; moderate-quality evidence). The most prominent bacterial pathogen in CAP in this age group is Streptococcus pneumoniae, and amoxicillin provides coverage against this organism. Azithromycin would be an appropriate choice in an older child because Mycoplasma pneumoniae would be more common. Moxifloxacin should not be used in children. Ceftriaxone and cefdinir can both be used to treat CAP, but they are broader spectrum antibiotics and would not be a first-line choice in this age group.

A 52-year-old male presents with a swollen and tender area anterior to the left ear and extending to below the left angle of the mandible. One week ago he had a Nissen fundoplication for intractable GERD. This was complicated by difficulty swallowing and drinking. On examination his tympanic temperature is 37.7°C (99.9°F), his blood pressure is 110/70 mm Hg, and his pulse rate is 95 beats/min and regular. His left parotid gland is diffusely enlarged and tender. Purulent material is noted coming from the left parotid duct orifice. Which one of the following would be most appropriate at this point? A. Amoxicillin/clavulanate (Augmentin) B. Penicillin C. CT of the parotid gland D. Incision and drainage of the parotid gland E. Excision of the parotid gland

A. Amoxicillin/clavulanate (Augmentin) This case is typical for acute parotitis, which is commonly caused by dehydration and can be diagnosed from the history and examination. Empiric treatment is directed toward gram-positive and anaerobic organisms, with the most common pathogen being Staphylococcus. These are often penicillin resistant so a β-lactamase inhibitor is the agent of choice. Treatment should be followed up with cultures. Administration of sialagogues such as lemon drops may be helpful, as well as parotid gland massage. CT or MRI may help confirm the diagnosis but imaging is usually not necessary. The history and clinical examination are most important for making the diagnosis. Incision and drainage would be appropriate only for an abscess, and surgical removal of the parotid gland is not indicated.

A 52-year-old female with a 60-pack-year history of cigarette smoking and known COPD presents with a 1-week history of increasing purulent sputum production and shortness of breath on exertion. Which one of the following is true regarding the management of this problem? A. Antibiotics should be prescribed B. Intravenous corticosteroids are superior to oral corticosteroids C. Inhaled corticosteroids should be started or the dosage increased D. Levalbuterol (Xopenex) is superior to albuterol E. Acetylcysteine should be given if the patient is hospitalized

A. Antibiotics should be prescribed Antibiotic use in moderately or severely ill patients with a COPD exacerbation reduces the risk of treatment failure or death, and may also help patients with mild exacerbations. Brief courses of systemic corticosteroids shorten hospital stays and decrease treatment failures. Studies have not shown a difference between oral and intravenous corticosteroids. Inhaled corticosteroids are not helpful in the management of an acute exacerbation. Levalbuterol and albuterol have similar benefits and adverse effects. Acetylcysteine, a mucolytic agent, has not been shown to be helpful for routine treatment of COPD exacerbations.

A 43-year-old female smoker has type 2 diabetes mellitus, morbid obesity, and a recent diagnosis of symptomatic peripheral arterial disease. You have started her on atorvastatin (Lipitor), offered a supervised exercise program, and discussed smoking cessation and interventions. Which one of the following should be recommended to prevent cardiovascular events in this patient? A. Aspirin B. Cilostazol (Pletal) C. Enoxaparin (Lovenox) D. Pentoxifylline E. Warfarin (Coumadin)

A. Aspirin Patients with symptomatic peripheral arterial disease should be started on a daily dose of either aspirin or clopidogrel to prevent cardiovascular events such as acute myocardial infarction or stroke (SOR B). Cilostazol is a phosphodiesterase inhibitor with both antiplatelet and arterial vasodilatory activity. It has been shown to improve claudication symptoms by 50% compared to placebo. Likewise, pentoxifylline is also used in the treatment of claudication symptoms but is less effective than cilostazol and is reserved as a second-line agent. Neither agent has been shown to decrease cardiovascular events in patients with symptomatic peripheral artery disease. Neither enoxaparin nor warfarin is indicated for symptomatic peripheral artery disease.

You respond to a code blue in the obstetrics department. The patient is a 19-year-old primigravida at 35 weeks gestation, hospitalized with severe preeclampsia. A nurse anesthetist has placed an oral airway and is administering 100% oxygen to the apneic patient. She reports no difficulty ventilating the patient with a bag and valve, and no gagging with oral airway insertion. The patient's blood pressure is 100/60 mm Hg and her pulse rate is 70 beats/min and regular. Her pupils are equal and sluggishly reactive, and she is flaccid and areflexic. The patient had been treated with a magnesium sulfate infusion and a recent bolus of labetalol. Which one of the following medications should you administer initially? A. Calcium gluconate B. Fosphenytoin C. Labetalol D. Lorazepam (Ativan) E. Dopamine

A. Calcium gluconate During the treatment of severe preeclampsia with intravenous magnesium, the occurrence of apnea and areflexia is most consistent with magnesium toxicity. In addition to hemodynamic support, calcium infusion is recommended as an antidote. Calcium chloride can be used if a central line has been established. Calcium gluconate would be safer with a peripheral intravenous site. Lorazepam, phenytoin, and fosphenytoin are less useful in preventing eclamptic seizures than magnesium. Labetalol is not indicated given the patient's current blood pressure level. Dopamine, a pressor agent, is not indicated in this scenario, and could aggravate the patient's preeclampsia.

A 28-year-old female just delivered a male infant over an intact perineum. She has had polyhydramnios during this pregnancy, but her prenatal course has otherwise been normal. Her only significant chronic medical problem is asthma, treated with a long-acting β-agonist/corticosteroid combination inhaler. Vital signs were stable throughout her labor. After delivery of the placenta, bleeding becomes brisk and you note a soft, boggy, uterus. Which one of the following medications is contraindicated in this patient? A. Carboprost (Hemabate) B. Methylergonovine C. Misoprostol (Cytotec) D. Oxytocin (Pitocin)

A. Carboprost (Hemabate) All of the drugs listed are appropriate for uterine atony and postpartum hemorrhage. Carboprost should not be used in this patient, however, as it is contraindicated in patients with asthma. Methylergonovine ismcontraindicated in hypertensive patients but may be used in patients with asthma.

A 62-year-old male comes to your office as a new patient. He has a past history of a myocardial infarction and is currently in stage C heart failure according to the American Heart Association classification. His ejection fraction is 30%. Which one of the following medications that the patient is currently taking is potentially harmful and should be discontinued if possible? A. Diltiazem (Cardizem) B. Lisinopril (Prinivil, Zestril) C. Carvedilol (Coreg) D. Atorvastatin (Lipitor)

A. Diltiazem (Cardizem) ACE inhibitors or angiotensin receptor blockers should be used in all patients with a history of myocardial infarction and reduced ejection fraction. Aldosterone receptor antagonists are indicated in patients who have a left ventricular ejection fraction <35%. Nondihydropyridine calcium channel blockers with negative inotropic effects (verapamil and diltiazem) may be harmful in patients with low left ventricular ejection fractions. Statin therapy is recommended in all patients with a history of myocardial infarction. Evidence-based B-blockers (carvedilol or metoprolol succinate) should be used in all patients with a history of myocardial infarction.

During rounds, you notice a new rash on a full-term 2-day-old white female. It consists of 1-mm pustules surrounded by a flat area of erythema, and is located on the face, trunk, and upper arms. An examination is otherwise normal, and she does not appear ill. Which one of the following is the most likely diagnosis? A. Erythema toxicum neonatorum B. Transient neonatal pustular melanosis C. Acne neonatorum D. Systemic herpes simplex E. Staphylococcus aureus sepsis

A. Erythema toxicum neonatorum This infant has the typical "flea-bitten" rash of erythema toxicum neonatorum (ETN). Transient neonatal pustular melanosis is most common in African-American newborns, and the lesions lack the surrounding erythema typical of ETN. Acne neonatorum is associated with closed comedones, mostly on the face. As the infant described is not ill, infectious etiologies are unlikely.

A 46-year-old female presents to your office with a 2-week history of pain in her left shoulder. She does not recall any injury, and the pain is present when she is resting and at night. Her only chronic medical problem is type 2 diabetes mellitus. On examination, she has limited movement of the shoulder and almost complete loss of external rotation. Radiographs of the shoulder are normal, as is her erythrocyte sedimentation rate. Which one of the following is the most likely diagnosis? A. Frozen shoulder B. Torn rotator cuff C. Impingement syndrome D. Chronic posterior shoulder dislocation E. Osteoarthritis

A. Frozen shoulder Frozen shoulder is an idiopathic condition that most commonly affects patients between the ages of 40 and 60. Diabetes mellitus is the most common risk factor for frozen shoulder. Symptoms include shoulder stiffness, loss of active and passive shoulder rotation, and severe pain, including night pain. Laboratory tests and plain films are normal; the diagnosis is clinical (SOR C). Frozen shoulder is differentiated from chronic posterior shoulder dislocation and osteoarthritis on the basis of radiologic findings. Both shoulder dislocation and osteoarthritis have characteristic plain film findings. A patient with a rotator cuff tear will have normal passive range of motion. Impingement syndrome does not affect passive range of motion, but there will be pain with elevation of the shoulder.

Which one of the following drugs is NOT effective for maintenance therapy in bipolar disorders? A. Haloperidol B. Lamotrigine (Lamictal) C. Lithium D. Quetiapine (Seroquel) E. Valproate sodium (Depacon)

A. Haloperidol Lithium, valproate, lamotrigine, and some antipsychotics (including quetiapine) are effective treatments for both acute depression and maintenance therapy of bipolar disorders. Haloperidol is an effective treatment for acute mania in bipolar disorders, but not for maintenance therapy or acute depression.

A 4-year-old white male is brought to your office because he has had a low-grade fever and decreased oral intake over the past few days. On examination you note shallow oral ulcerations confined to the posterior pharynx. Which one of the following is the most likely diagnosis? A. Herpangina B. Herpes C. Mononucleosis D. Roseola infantum E. Rubella

A. Herpangina Herpangina is a febrile disease caused by coxsackieviruses and echoviruses. Vesicles and subsequent ulcers develop in the posterior pharyngeal area (SOR C). Herpes infection causes a gingivostomatitis that involves the anterior mouth. Mononucleosis may be associated with petechiae of the soft palate, but does not usually cause pharyngeal lesions. The exanthem in roseola usually coincides with defervescence. Mucosal involvement is not noted. Rubella may cause an enanthem of pinpoint petechiae involving the soft palate (Forschheimer spots), but not the pharynx.

A 70-year-old male with widespread metastatic prostate cancer is being cared for through a local hospice. Surgery, radiation, and hormonal therapy have failed to stop the cancer, and the goal of his care is now symptom relief. Over the past few days he has been experiencing respiratory distress. His oxygen saturation is 94% on room air and his lungs are clear to auscultation. His respiratory rate is 16/min. Which one of the following would be best at this point? A. Morphine B. Oxygen C. Albuterol (Proventil, Ventolin) D. Haloperidol

A. Morphine Dyspnea is a frequent and distressing symptom in terminally ill patients. In the absence of hypoxia, oxygen is not likely to be helpful. Opiates are the mainstay of symptomatic treatment and other measures may be appropriate in specific circumstances. For example, inhaled bronchodilators or glucocorticoids may be helpful in patients with COPD, and diuresis may be helpful in patients with heart failure. The evidence for oxygen in patients with hypoxemia is not clear, but there is no benefit from oxygen for nonhypoxemic patients.

Which one of the following is the most common cause of unintentional deaths in children? A. Motor vehicle accidents B. Drowning C. Poisoning D. Fires E. Falls

A. Motor Vehicle Accidnets Unintentional injuries account for 40% of childhood deaths. Motor vehicle accidents are the most frequent cause of these deaths (58.2% of unintentional deaths). The proper use of child restraints is the most effective way to prevent injury or death, and the American Academy of Family Physicians and the American Academy of Pediatrics strongly recommend that physicians actively promote the proper use of motor vehicle restraints for all patients. Drowning accounts for 10.9% of all unintentional deaths in children, poisoning for 7.7%, fires 5.7%, and falls 1.4%.

You see a 5-year-old white female with in-toeing due to excessive femoral anteversion. She is otherwise normal and healthy, and her mobility is unimpaired. Her parents are greatly concerned with her appearance and possible future disability, and request that she be treated. You recommend which one of the following? A. Observation B. Medial shoe wedges C. Torque heels D. Sleeping in a Denis Browne splint for 6 months E. Derotational osteotomy of the femur

A. Observation There is little evidence that femoral anteversion causes long-term functional problems. Studies have shown that shoe wedges, torque heels, and twister cable splints are not effective. Surgery should be reserved for children 8-10 years of age who still have cosmetically unacceptable, dysfunctional gaits. Major complications of surgery occur in approximately 15% of cases, and can include residual in-toeing, out-toeing, avascular necrosis of the femoral head, osteomyelitis, fracture, valgus deformity, and loss of position. Thus, observation alone is appropriate for a 5-year-old with uncomplicated anteversion.

A 17-year-old male presents to the urgent care clinic 15 minutes after being stung by a wasp. He feels weak, his voice is hoarse, and he is beginning to have trouble breathing. Which one of the following should be administered first? A. Oral antihistamines B. An oral leukotriene-receptor antagonist C. Intranasal antihistamines D. Intranasal corticosteroids E. Furnace filters and mite-proof bedding covers

A. Oral antihistamines H1 and H2 histamine blockers and corticosteroids may be useful, but they are not first-line treatments for an anaphylactic reaction to a Hymenoptera sting. Intravenous normal saline may also be necessary for fluid resuscitation, but the first treatment should be immediate administration of intramuscular epinephrine.

A 25-year-old male presents with a 3-day history of cough, chills, and fever. The patient was previously healthy and has no chronic medical problems. He has no known drug allergies. On examination he is alert and oriented, and has a temperature of 38.4°C (101.1°F), a pulse rate of 88 beats/min, a blood pressure of 120/70 mm Hg, a respiratory rate of 16/min, and an oxygen saturation of 98%. Auscultation of the lungs reveals no wheezing and the presence of right basilar crackles. A chest radiograph shows a right lower lobe infiltrate. There is a low rate of macrolide-resistant pneumococcus in the community. Which one of the following is the most appropriate initial management of this patient? A. Outpatient treatment with azithromycin (Zithromax) B. Outpatient treatment with cefuroxime (Ceftin) C. Inpatient treatment on the medical floor with ceftriaxone (Rocephin) and azithromycin D. Inpatient treatment on the medical floor with piperacillin/tazobactam (Zosyn) and levofloxacin E. Inpatient treatment in the intensive-care unit with ceftriaxone, levofloxacin, and vancomycin (Vancocin)

A. Outpatient treatment with azithromycin (Zithromax) In patients with community-acquired pneumonia it is necessary to decide on both the antibiotic regimen and the treatment setting. The decision regarding site of care is based on the severity of illness, which can be assessed with tools such as the CURB-65 score, which take into account factors such as respiratory rate, blood pressure, uremia, confusion, and age. Patients who have only mild symptoms can be treated with azithromycin on an outpatient basis if there is a low level of macrolide resistance in the community. If there is a high level of resistance in the community, if the patient has comorbidities such as diabetes mellitus or COPD, or if there is a history of use of an immunosuppressing drug or recent use of an antibiotic, the patient can still be treated as an outpatient but should be treated with levofloxacin. Patients with more severe symptoms, such as an elevated pulse rate or respiratory rate, should be treated on an inpatient basis with ceftriaxone or azithromycin. Patients who have more severe symptoms along with bronchiectasis should be treated with piperacillin/tazobactam plus levofloxacin. Patients with the most severe symptoms, including hypotension, a more elevated pulse rate, low oxygen saturation, and confusion, should be treated in the intensive-care unit with levofloxacin and vancomycin.

A 61-year-old female is found to have a serum calcium level of 11.6 mg/dL (N 8.6-10.2) on routine laboratory screening. To confirm the hypercalcemia you order an ionized calcium level, which is 1.49 mmol/L (N 1.14-1.32). Additional testing reveals an intact parathyroid hormone level of 126 pg/mL (N 15-75) and a urine calcium excretion of 386 mg/24 hr (N 100-300). Which one of the following is the most likely cause of the patient's hypercalcemia? A. Primary hyperparathyroidism B. Malignancy C. Familial hypocalciuric hypercalcemia D. Hypoparathyroidism E. Hyperthyroidism

A. Primary hyperparathyroidism Primary hyperparathyroidism and malignancy account for more than 90% of hypercalcemia cases. These conditions must be differentiated early to provide the patient with optimal treatment and an accurate prognosis. Humoral hypercalcemia of malignancy implies a very limited life expectancy—often only a matter of weeks. On the other hand, primary hyperparathyroidism has a relatively benign course. Intact parathyroid hormone (PTH) will be suppressed in cases of malignancy-associated hypercalcemia, except for extremely rare cases of parathyroid carcinoma. Thyrotoxicosis-induced bone resorption elevates serum calcium, which also results in suppression of PTH. Patients with familial hypocalciuric hypercalcemia (FHH) have moderate hypercalcemia but relatively low urinary calcium excretion. PTH levels can be normal or only mildly elevated despite the hypercalcemia. This mild elevation can lead to an erroneous diagnosis of primary hyperparathyroidism. The conditions can be differentiated by a 24-hour urine collection for calcium; calcium levels will be high or normal in patients with hyperparathyroidism and low in patients with FHH.

A 30-year-old female stepped off a curb earlier today and twisted her left ankle. She was able to bear weight immediately following the injury and tried to continue her normal routine, but the pain in her ankle and foot increased over the next few hours. She comes to your office and your examination reveals swelling of the ankle and bruising of the lateral foot. Tenderness to palpation is present over the distal aspect of the fibula and lateral malleolus and to a lesser degree over the proximal fifth metatarsal. No bony tenderness is present along the medial aspect of the ankle or foot. According to the Ottawa Ankle Rules, which one of the following would be most appropriate at this point? A. Radiographs of the ankle and foot B. Radiographs of the foot only C. Radiographs of the ankle only D. No radiographs

A. Radiographs of the ankle and foot The Ottawa Ankle Rules are widely accepted guidelines for appropriate evaluation of ankle and midfoot injuries occurring in adults age 19 or older presenting for the first time in a clinical setting. The guidelines utilize the historical and physical findings to determine which radiographic studies, if any, are indicated. Patients who were able to bear weight immediately following their injury and who can take 4 steps independently in a clinical setting require radiographic study only when the following criteria are met: pain is present in the malleolar zone and bony tenderness of the posterior edge or tip of either malleolus is elicited (ankle radiograph), or pain is present in the midfoot zone and bony tenderness of either the base of the fifth metatarsal or the navicular region is present.

A 75-year-old male develops a mild Clostridium difficile infection and is treated with 10 days of metronidazole (Flagyl), 500 mg orally 3 times daily. The diarrhea recurs 10 days after he completes the course of treatment. Which one of the following would be most appropriate? A. Repeat the course of metronidazole B. Repeat the course of metronidazole and add vancomycin C. Administer vancomycin intravenously D. Prescribe loperamide (Imodium), 4 mg twice daily as needed E. Prescribe a probiotic

A. Repeat the course of metronidazole Clostridium difficile infection is more common with aging and can be treated with either metronidazole or vancomycin daily. For mild recurrent disease, repeating the course of the original agent is appropriate (SOR B). Multiple recurrences or severe disease warrants the use of both agents. The effectiveness of probiotics such as Lactobacillus remains uncertain. Intravenous vancomycin has not been effective. Antiperistaltic drugs should be avoided.

A 24-year-old female presents with pelvic pain. She says that the pain is present on most days, but is worse during her menses. Ibuprofen has helped in the past but is no longer effective. Her menses are normal and she has only one sexual partner. A physical examination is normal. Which one of the following should be the next step in the workup of this patient? A. Transvaginal ultrasonography B. CT of the abdomen and pelvis C. MRI of the pelvis D. A CA-125 level E. Colonoscopy

A. Transvaginal ultrasonography The initial evaluation for chronic pelvic pain should include a urinalysis and culture, cervical swabs for gonorrhea and Chlamydia, a CBC, an erythrocyte sedimentation rate, a β-hCG level, and pelvic ultrasonography. CT and MRI are not part of the recommended initial diagnostic workup, but may be helpful in further assessing any abnormalities found on pelvic ultrasonography. Referral for diagnostic laparoscopy is appropriate if the initial workup does not reveal a source of the pain, or if endometriosis or adhesions are suspected. Colonoscopy would be indicated if the history or examination suggests a gastrointestinal source for the pain after the initial evaluation.

About a month after returning from the Middle East, an American soldier develops a papule on his forearm that subsequently ulcerates to form a shallow annular lesion with a raised margin. The lesion shows no signs of healing 3 months after it first appeared. He has no systemic symptoms. The most likely diagnosis is: A. leishmaniasis B. schistosomiasis C. malaria D. trypanosomiasis E. syphilis

A. leishmaniasis The indolent course of the sore described favors the diagnosis of cutaneous leishmaniasis. Neither malaria nor schistosomiasis produces these sores. The chancres of syphilis and trypanosomiasis are more fleeting in duration.

A 63-year-old male with type 2 diabetes mellitus is seen in the emergency department for an acute, superficial, previously untreated infected great toe. Along with Staphylococcus aureus, which one of the following is the most common pathogen in this situation? A. Pseudomonas B. Streptococcus C. Clostridium D. Escherichia coli E. Adenovirus

B. Streptococcus The most common pathogens in previously untreated acute superficial foot infections in diabetic patients are aerobic gram-positive Staphylococcus aureus and β-hemolytic streptococci (groups A, B, and others). Previously treated and deep infections are often polymicrobial.

A 6-year-old male is brought to your office for a well child check. His vital signs are normal; he is 117 cm (46 in) tall and weighs 19 kg (42 lb). The patient has grown out of his car seat, and his mother recently was told by a friend that he can now sit in the front seat of a car. She asks you if this is true. According to recommendations from the American Academy of Pediatrics, which one of the following is true for this patient? A. He should be using a rear-facing child safety seat in the back seat of the car B. He should be using a belt-positioning booster seat in the back seat of the car C. He should be using a belt-positioning booster seat and can ride in the front or back seat of the car D. He no longer needs a safety seat but should always ride in the back seat of the car E. He no longer needs a safety seat and can ride in the front or back seat of the car

B. Children who reach the weight or height limit of their forward-facing child safety seat should use a belt-positioning booster seat until the seat belt fits properly, typically when the child is 145 cm (57 in) tall and between 8 and 12 years of age (SOR C). All children younger than 13 years should ride in the rear seats of vehicles for optimal protection (SOR C). All infants and toddlers should ride in a rear-facing child safety seat until they are 2 years of age or until they reach the weight or height limit recommended by the seat's manufacturer (SOR C).

Which one of the following is associated with treatment of COPD with inhaled corticosteroids? (check one) A. An increased risk of monilial vaginitis B. An increased risk of bruising C. Consistent improvement in FEV1 D. A decreased risk of pneumonia E. Decreased mortality

B. Inhaled corticosteroids increase the risk of bruising, candidal infection of the oropharynx, and pneumonia. They also have the potential for increasing bone loss and fractures. They decrease the risk of COPD exacerbations but have no benefit on mortality and do not improve FEV1 on a consistent basis.

A 42-year-old female presents with a cough productive of blood-streaked sputum for the past 3 days. Her hemoptysis was preceded by several days of rhinorrhea, congestion, and subjective fever. She estimates the total amount of blood loss to be approximately 1 tablespoon. She is a nonsmoker and her past medical history is unremarkable. Vital signs are within normal limits, and other than an intermittent cough there are no abnormal findings on the physical examination. Which one of the following would be the most appropriate next step? A. Observation B. A chest radiograph C. Chest CT D. Bronchoscopy E. Antibiotics

B. A chest radiograph The first step in the evaluation of nonmassive hemoptysis is to obtain a chest radiograph. If this is normal and there is a high risk of malignancy (patient age 40 years or older with at least a 30-pack year smoking history), chest CT should be ordered. Bronchoscopy should also be considered in the workup of high-risk patients. If a chest radiograph shows an infiltrate, treatment with antibiotics is warranted. If the chest radiograph is normal the patient is at low risk for malignancy, and if the history does not suggest lower respiratory infection and hemoptysis does not recur, observation can be considered.

Which one of the following is most likely to be seen with diastolic dysfunction? A. A dialated left ventricle B. A preserved ejection fraction C. Aortic insufficiency D. Pericardial effusion

B. A preserved ejection fraction Heart failure due to diastolic dysfunction occurs in the older population. The criteria for diastolic heartfailure include symptoms and signs consistent with heart failure (including dyspnea), a nondilated leftventricle with a preserved ejection fraction (³50%), and evidence of structural heart disease such as diastolic dysfunction on echocardiography

A 45-year-old male with diabetes mellitus returns to your office for follow-up. He is on metformin (Glucophage), 1000 mg/day, as well as atorvastatin (Lipitor), 40 mg daily for hyperlipidemia. There is no diagnosis of hypertension, and his blood pressure at today's visit is 120/70 mm Hg. Laboratory results include a hemoglobin A1c of 6.4% and an LDL-cholesterol level of 105 mg/dL. His urine albumin/creatinine ratio is in the microalbuminuric range for the first time. Which one of the following would be most appropriate at this point? A. Renal ultrasonography B. A repeat urine albumin/creatinine ratio C. 24-hour urine for microalbumin D. Increasing the atorvastatin dosage E. Stopping metformin

B. A repeat urine albumin/creatinine ratio This normotensive diabetic patient, appropriately screened for microalbuminuria, should have this finding confirmed on at least one of two additional spot tests, since temporary factors other than nephropathy can also result in microalbuminuria. Once a diagnosis of chronic kidney disease is confirmed, renal ultrasonography should be ordered to detect potentially reversible causes. A 24-hour urine is not necessary since the urine microalbumin/creatinine ratio correlates well with a 24-hour urine for albumin. Metformin is not contraindicated in the presence of microalbuminuria alone without a decline in the glomerular filtration rate. The patient is already on high-intensity statin therapy and there is no specific indication to increase the statin dosage based on his current LDL-cholesterol level since treatment to the target LDL-cholesterol goal has fallen out of favor.

You evaluate an 18-month-old male with fecal impaction and determine that disimpaction is indicated. Which one of the following would be most appropriate initially? A. An oral stimulant such as sennosides (Senokot) B. An oral osmotic agent such as polyethylene glycol 3350 (MiraLax) C. An enema using saline, mineral oil, or phosphate soda D. A bisacodyl (Dulcolax) rectal suppository E. Manual disimpaction

B. An oral osmotic agent such as polyethylene glycol 3350 (MiraLax) Oral osmotics such as polyethylene glycol-based solutions are recommended as an appropriate initial approach to constipation in children because they are effective, easy to administer, noninvasive, and well tolerated (SOR C). Rectal therapies are similar in terms of effectiveness but are more invasive and less commonly used as first-line treatment (SOR A). Oral stimulants and bisacodyl rectal suppositories are not recommended for children under 2 years of age. Enemas are sometimes used as second-line therapy, but the addition of enemas to oral laxative regimens does not improve outcomes in children with severe constipation (SOR B). Manual disimpaction is a more invasive option and is not recommended as first-line treatment in young children.

Which one of the following is most appropriate for patients with asplenia? A. Lifelong daily antibiotic prophylaxis B. Antibiotics for any episode of fever C. An additional dose of Hib vaccine D. Avoiding live attenuated influenza vaccine E. Withholding pneumococcal vaccine

B. Antibiotics for any episode of fever Asplenic patients who develop a fever should be given antibiotics immediately. Due to the increased risk of pneumococcal sepsis in asplenic patients, vaccinations against these particular bacteria are specifically recommended. Since pneumococcal conjugate vaccine (PCV13) and pneumococcal polysaccharide vaccine (PPSV23) can interact with each other they should be given at least 8 weeks apart. Prophylactic penicillin given orally twice a day is particularly important in children under 5 years of age who are asplenic, and may be considered for 1-2 years post splenectomy in older patients. Lifelong daily antibiotics may be considered following post-splenectomy sepsis. The risk for Haemophilus influenzae type b infection is not increased in asplenic patients, so additional vaccine is not needed for those who have already been vaccinated. Live attenuated influenza vaccine may be used in asplenic patients, unless they have sickle cell disease.

23-year-old healthy male is sexually active with other men and does not use condoms. He is interested in reducing his risk of contracting HIV by using a daily oral antiretroviral medication. Which one of the following laboratory tests should be done no more than 7 days before initially prescribing pre-exposure prophylaxis with emtricitabine/tenofovir disoproxil (Truvada)? A. A CD4 cell count B. Antibody testing for HIV C. Hemoglobin concentration D. A platelet count E. An ALT level

B. Antibody testing for HIV AIt is of critical importance that patients have a documented negative HIV antibody test (from serum or point-of-care fingerstick) prior to starting pre-exposure prophylaxis (PrEP) to avoid inadvertent treatment of HIV infection with emtricitabine/tenofovir. This is the only medication currently approved in the United States for PrEP, but it is inadequate for HIV treatment. Using this treatment by itself in HIV-positive patients increases the risk of HIV strains developing resistance to these antiviral agents. Other recommended testing prior to PrEP use includes creatinine clearance calculation, hepatitis B antibody testing, screening for sexually transmitted diseases, and pregnancy testing in females capable of pregnancy. The CDC does not recommend testing liver function, hemoglobin, or platelet levels prior to PrEP use in otherwise healthy individuals.

A 68-year-old male with end-stage lung cancer is being treated for pain secondary to multiple visceral and skeletal metastases. He has been on oral ibuprofen and parenteral morphine. However, over the past few weeks he reports progressive worsening of his pain. In order to achieve better pain control his morphine dosage has been continuously titrated up. In spite of this increase he continues to report severe pain that is now diffuse and occurs even when his caregivers touch him. Which one of the following would be most appropriate at this time? A. Increase the morphine dosage until continuous sedation is obtained B. Attempt a reduction in the morphine dosage C. Add an anxiolytic to help relieve anxiety D. Advise the family that nothing more can be done for his pain

B. Attempt a reduction in the morphine dosage Opioid-induced hyperalgesia is characterized by a paradoxical increase in sensitivity to pain despite an increase in the opioid dosage. It is seen in patients who are receiving high doses of parenteral opioids such as morphine. Patients report the development of diffuse pain away from the site of the original pain. Allodynia, a perception of pain in the absence of a painful stimulus, is also typical in opioid-induced hyperalgesia. Strategies to manage this condition include reducing the current opioid dosage, and occasionally eliminating the current opioid and starting another opioid. The addition of non-opioid pain medications should also be considered. The addition of an anxiolytic is not likely to improve this patient's pain

A 22-year-old female with a 2-week history of paroxysmal cough is found to have pertussis confirmed by a polymerase chain reaction test and a nasal swab culture. Which one of the following is the antibiotic of choice for this patient? A. Amoxicillin B. Azithromycin (Zithromax) C. Ciprofloxacin (Cipro) D. Clindamycin (Cleocin) E. Doxycycline

B. Azithromycin (Zithromax) Azithromycin should be considered the preferred agent for the treatment and prophylaxis of pertussis (SOR A). Trimethoprim/sulfamethoxazole is an alternative in cases of allergy or intolerance to macrolides. Because of the possibility of treatment benefit, and because of the potential of antibiotics to decrease transmission, the CDC continues to recommend antibiotics for the treatment of pertussis. In order to prevent transmission of the infection, treatment should be initiated within 6 weeks of the onset of cough in patients younger than 12 months, and within 3 weeks in all other patients.

You see a 55-year-old female for the first time. She has a 2-year history of chronic daily cough; thick, malodorous sputum; and occasional hemoptysis. She has been treated with antibiotics for recurrent respiratory infections, but is frustrated with her continued symptoms. She has never smoked. Her FEV1/FVC ratio is 60% and CT shows bronchial wall thickening and luminal dilation. The most likely diagnosis is? A. Emphysema B. Bronchiectasis C. Chronic bronchitis D. Bronchiolitis E. Asthma

B. Bronchiectasis Bronchiectasis is an illness of the bronchi and bronchioles involving obstructive and infectious processes that injure airways and cause luminal dilation. In addition to daily viscid, often purulent sputum production with occasional hemoptysis, wheezing and dyspnea occur in 75% of patients. Emphysema and chronic bronchitis, forms of COPD, also cause a decreased FEV1/FVC ratio, but the sputum is generally mucoid and luminal dilation of bronchi is not characteristically present. Bronchiolitis is usually secondary to respiratory syncytial virus infection in young children. Asthma is not characterized by the sputum and CT findings seen in this patient.

Which one of the following conditions can affect hemoglobin A1c levels? A. Heart failure B. Chronic hemolytic anemia C. COPD D. Hypothyroidism

B. Chronic hemolytic anemia The hemoglobin A1c (HbA1c) blood test provides information regarding average glucose levels over the past 3 months. Any condition that shortens erythrocyte survival or decreases mean erythrocyte age, such as recent acute blood loss or hemolytic anemia, will falsely lower HbA1c levels. Hemoglobin variants and iron deficiency, kidney failure, and liver disease can also affect HbA1c results. Heart failure, COPD, and hypothyroidism do not influence HbA1c values.

A 30-year-old female presents with dysuria and flank pain. She reports a fever of 102°F yesterday morning. She has not taken any antipyretics since that time, and today her temperature is 36.7°C (98.1°F). She has a pulse rate of 93 beats/min, a respiratory rate of 16/min, and a blood pressure of 116/58 mm Hg. The remainder of her physical examination is unremarkable, except for marked costovertebral angle tenderness. A CBC reveals a WBC count of 14,590/mm3 (N 4300-10,800) with 85% neutrophils, 12% lymphocytes, and 3% basophils, but is otherwise normal. A urine $-hCG is negative. A urine dipstick is positive for leukocyte esterase, and urine microscopic analysis is notable for <1 RBC and >50 WBCs/hpf. Urine culture results are pending. You confirm she has no medication allergies. Which one of the following oral antibiotics would be most appropriate for empiric therapy? A. Amoxicillin B. Ciprofloxacin (Cipro) C. Erythromycin D. Metronidazole (Flagyl) E. Nitrofurantoin (Furadantin)

B. Ciprofloxacin (Cipro) Acute pyelonephritis is a common bacterial infection of the renal pelvis and kidney most often seen in young adult women. It is most commonly caused by Escherichia coli. Outpatient treatment with oral antibiotics is safe in most adults with mild or moderate pyelonephritis (SOR B). An oral fluoroquinolone such as ciprofloxacin is usually the first-line therapy in mild and moderate cases in areas where the rate of fluoroquinolone resistance in E. coli is <10% (SOR A). If the community fluoroquinolone resistance rate exceeds 10%, a one-time dose of a parenteral antimicrobial such as ceftriaxone or a consolidated dose of an aminoglycoside should be given, followed by an oral fluoroquinolone regimen (SOR B). Alternative oral agents include bactrim and B-lactam antibiotics; however, these are not first-line empiric agents, due to high levels of resistance (SOR A), and should not be used for treatment until the uropathogen is confirmed to be susceptible. Amoxicillin and nitrofurantoin are sometimes used to treat uncomplicated cystitis but these agents are less effective than other available agents for treatment of pyelonephritis (SOR B). Erythromycin and metronidazole are not appropriate for treating pyelonephritis.

Which one of the following conditions is the leading cause of death for patients with rheumatoid arthritis? A. Accidents B. Coronary artery disease C. Thromboembolic disease D. Lymphoma E. Lung cancer

B. Coronary artery disease As is true for the general population in the United States, coronary artery disease is the leading cause of death in patients with rheumatoid arthritis (RA). RA patients have accelerated atherosclerosis related to a chronic inflammatory state. It is thus particularly important to address modifiable risk factors for coronary disease in these patients, including tobacco use, hypertension, and dyslipidemia. Patients with RA also have an increased risk of lymphoma, lung cancer, and thromboembolic disease, but these are not as common as coronary disease. Infections are a concern for patients on disease-modifying agents but are not the leading cause of death.

A 24-year-old female with a 2-year history of dyspnea on exertion has been diagnosed with exercise-induced asthma by another physician. Which one of the following findings on pulmonary function testing would raise concerns that she actually has vocal cord dysfunction? A. A good response to an inhaled β-agonist B. Flattening of the inspiratory portion of the flow-volume loop, but a normal expiratory phase C. Flattening of the expiratory portion of the flow-volume loop, but a normal inspiratory phase D. Flattening of both the inspiratory and expiratory portion of the flow-volume loop E. A decreased FEV1 and a normal FVC

B. Flattening of the inspiratory portion of the flow-volume loop, but a normal expiratory phase The diagnosis of vocal cord dysfunction should be considered in patients diagnosed with exercise-induced asthma who do not have a good response to β-agonists before exercise. Pulmonary function testing with a flow-volume loop typically shows a normal expiratory portion but a flattened inspiratory phase (SOR C). A decreased FEV1 and normal FVC would be consistent with asthma.

A 45-year-old male presents with a 3-month history of hoarseness. He denies any other complaints and has not been ill recently. He is not on any medication, has no history of chronic medical problems, and does not smoke cigarettes or drink alcohol. Which one of the following would be the most appropriate management of this patient? A. Voice rest for 1 month B. Laryngoscopy C. A trial of a proton pump inhibitor D. A trial of inhaled corticosteroids E. Oral corticosteroids

B. Laryngoscopy Laryngoscopy should be performed to visualize the larynx and evaluate for vocal cord pathology in a patient whose hoarseness does not resolve within 3 months (SOR C). If a serious condition is suspected for some other reason, laryngoscopy should be performed regardless of the duration of symptoms. If there is a recent history of upper respiratory infection or vocal abuse, then it would be appropriate to recommend voice rest for 2 weeks. Laryngoscopy would then be indicated if the hoarseness did not improve or recurred after voice rest. For patients with symptoms of gastroesophageal reflux, a trial of a proton pump inhibitor is recommended (SOR B). Inhaled corticosteroids, especially fluticasone, may cause hoarseness. Oral corticosteroids do not have a role in the management of hoarseness.

A 77-year-old male presents with significant postherpetic neuralgia in a chest wall distribution. Which one of the following is most likely to be effective in diminishing his discomfort? A. Oral valacyclovir (Valtrex) B. Topical lidocaine (Xylocaine) patches C. Thoracic epidural corticosteroid injections D. Herpes zoster vaccine E. Acupuncture

B. Topical lidocaine (Xylocaine) patches Antiviral drugs are useful for treatment of acute herpes zoster but not for treatment of postherpetic neuralgia. Herpes zoster vaccine can prevent postherpetic neuralgia by reducing the incidence of herpes zoster but it has no role in the treatment of neuralgia. Neither acupuncture nor epidural corticosteroid injections are helpful in treating postherpetic neuralgia. Topical agents such as lidocaine patches and capsaicin cream or patches have been shown to reduce symptoms of postherpetic neuralgia, as have the oral agents gabapentin, pregabalin, and amitriptyline.

A previously healthy 74-year-old male presents to the emergency department with a fever and altered mental status. His illness began 2 days ago with symptoms of fever, malaise, body aches, reduced appetite, nausea, and diarrhea. His temperature is 39.6°C (103.3°F) in the emergency department and his examination is nonfocal. Initial laboratory studies include a sodium level of 131 mEq/L (N 135-145) and a WBC count of 14,200/mm3 (N 4500-11,000) with a neutrophilic predominance. Blood and urine cultures are obtained and he is admitted to the hospital for observation. The next morning he develops a productive cough and shortness of breath. You order a chest radiograph, which shows patchy consolidation of the bilateral bases. Which one of the following is the most likely cause of this patient's condition? A. Chlamydophila pneumoniae B. Legionella pneumophila C. Mycoplasma pneumoniae D. Streptococcus pneumoniae

B. Legionella pneumophila Pneumonia caused by Legionella pneumophila is commonly preceded by nonspecific systemic symptoms that may lead a clinician to consider other diagnoses. Symptoms may include high-grade fever, malaise, myalgias, anorexia, and headache. Gastrointestinal and neurologic symptoms are also common and include nausea, vomiting, abdominal pain, diarrhea, and confusion. Focal neurologic signs are less common, but have been reported. Localizing respiratory symptoms will typically develop later, most often a dry cough and dyspnea. From this point on the illness resembles a typical pneumonia with fever, productive cough, pleuritic pain, and breathlessness.

Which one of the following is recommended in all patients with croup, including those with mild disease? A. Humidification therapy B. Oral dexamethasone as a single dose C. Oral diphenhydramine (Benadryl) every 6 hours until improvement D. Subcutaneous epinephrine as a single dose E. Intramuscular ceftriaxone (Rocephin) as a single dose

B. Oral dexamethasone as a single dose A single dose of dexamethasone (0.15-0.60 mg/kg, usually given orally) is recommended in all patients with croup, including those with mild disease. Humidification therapy has not been proven beneficial. Nebulized epinephrine is an accepted treatment in patients with moderate to severe croup. Subcutaneous epinephrine, diphenhydramine, and ceftriaxone are not recommended treatments.

A 58-year-old male with COPD presents with a 5-day history of increased dyspnea and purulent sputum production. He is afebrile. His respiratory rate is 24/min, heart rate 90 beats/min, blood pressure 140/80 mm Hg, and oxygen saturation 90% on room air. Breath sounds are equal, and diffuse bilateral rhonchi are noted. He is currently using albuterol/ipratropium by metered-dose inhaler three times daily. In addition to antibiotics, which one of the following would be most appropriate for treating this exacerbation? A. A single dose of intramuscular dexamethasone B. Oral prednisone for 5 days C. Daily inhaled fluticasone (Flovent) D. Hospital admission for intravenous methylprednisolone sodium succinate (Solu-Medrol) E. No corticosteroids at this time

B. Oral prednisone for 5 days his patient most likely has a mild to moderate COPD exacerbation. His vital signs do not indicate a serious condition at this time, so he can be treated as an outpatient. Since he is already on a reasonable dose of an inhaled bronchodilator/ anticholinergic combination, he should be treated with an oral antibiotic and an oral corticosteroid. Intravenous corticosteroids offer no advantages over oral therapy, provided there are no gastrointestinal tract limitations such as poor motility or absorption. Oral corticosteroid therapy initiated early in a COPD exacerbation reduces the rate of treatment failure, decreases hospitalization rates, improves hypoxia and pulmonary function, and shortens the length of stay for patients requiring hospitalization. Short courses of oral corticosteroids (5-7 days) are as effective as longer ones (SOR A). Inhaled corticosteroids are ineffective in the treatment of a COPD exacerbation. Intramuscular dexamethasone has no role in treating COPD.

A 37-year-old graphic designer presents to your office with a history of several months of radial wrist pain. She does not recall any specific trauma but notes that it hurts to hold a coffee cup. Finkelstein's test is positive and a grind test is negative, and there is tenderness to palpation over the radial tubercle. Which one of the following would be most appropriate at this point? A. Plain radiography focusing on the scaphoid B. Rest and a thumb spica wrist splint C. MRI of the wrist D. A short arm cast

B. Rest and a thumb spica wrist splint This patient has de Quervain's tenosynovitis. Finkelstein's test has good sensitivity and specificity (SOR C) in patients with a negative grind test. A positive grind test would be more consistent with scaphoid fracture. A hand radiograph with secondary thumb spica splinting would be appropriate for a suspected scaphoid fracture, but the insidious onset as opposed to overt trauma makes this diagnosis unlikely in this case. A short arm cast is not indicated in de Quervain's tenosynovitis but may be appropriate for forearm/wrist fractures.

When treating acute adult asthma in the emergency department, using a metered-dose inhaler (MDI) with a spacer has been shown to result in which one of the following, compared to use of a nebulizer? A. Higher hospitalization rates B. Shorter stays in the emergency department C. Higher relapse rates D. Less improvement in peak-flow rates E. Increases in the total dose of albuterol

B. Shorter stays in the emergency department Compared to nebulizers, MDIs with spacers have been shown to lower pulse rates, provide greater improvement in peak-flow rates, lead to greater improvement in arterial blood gases, and decrease required albuterol doses. They have also been shown to lower costs, shorten emergency department stays, and significantly lower relapse rates at 2 and 3 weeks compared to nebulizers. There is no difference in hospital admission rates.

A 34-year-old white female sees you for a routine follow-up visit. She takes haloperidol, 2 mg after each meal, for schizophrenia, and you notice that she seems unable to sit still and is extremely anxious. The most likely cause of her restlessness is A. drug-induced parkinsonism B. akathisia C. tardive dyskinesia D. hysteria E. dystonia

B. akathisia Motor side effects of the antipsychotic drugs can be separated into five general categories: dystonias, parkinsonism, akathisia, withdrawal dyskinesias, and tardive dyskinesia. Akathisia is a syndrome marked by motor restlessness. Affected patients commonly complain of being inexplicably anxious, of being unable to sit still or concentrate, and of feeling comfortable only when moving. Hysteria is no longer considered a useful term.

A 40-year-old male respiratory therapist presents for a health examination prior to hospital employment. His history indicates that as a child he lived on a farm in Iowa. His examination is unremarkable, but a chest radiograph shows that both lung fields have BB-sized calcifications in a miliary pattern. No other findings are noted. A PPD skin test is negative. The findings in this patient are most likely a result of A. HIV infection B. histoplasmosis C. coccidioidomycosis D. tuberculosis E. cryptococcosis

B. histoplasmosis Asymptomatic patients in excellent health often present with this characteristic chest radiograph pattern, which is usually due to histoplasmosis infection, especially if the patient has been in the midwestern United States. Exposure to bird or bat excrement is a common cause, and treatment is usually not needed. This pattern is not characteristic of the other infections listed, although miliary tuberculosis is a remote possibility despite the negative PPD skin test.

A 79-year-old female had a total knee replacement yesterday. She has mild dementia as a result of a stroke 10 years ago, but her dementia has been stable since then. Last night she became confused and agitated, striking out at nurses, and could not be consoled. Which one of the following would be most appropriate at this time? A. Soft restraints B. CT of the head C. Adequate pain control D. A sedating SSRI such as paroxetine (Paxil) E. Lorazepam (Ativan) intravenously as needed

C. Adequate pain control This patient has postoperative delirium, which is associated with an increased mortality rate. Reorientation and pain management are important management strategies. Benzodiazepines, antipsychotics, antidepressants, and restraints are not helpful and may make the situation worse. Imaging modalities are not helpful in the absence of localizing signs.

A critically ill adult male is admitted to the intensive-care unit because of sepsis. He has no history of diabetes mellitus, but his glucose level on admission is 215 mg/dL and insulin therapy is ordered. Which one of the following is the most appropriate target glucose range for this patient? A. 80-120 mg/dL B. 100-140 mg/dL C. 120-160 mg/dL D. 140-180 mg/dL E. 160-200 mg/dL

C. 120-160 mg/dL The 2009 consensus guidelines on inpatient glycemic control issued by the American Association of Clinical Endocrinologists and the American Diabetes Association recommend insulin infusion with a target glucose level of 140-180 mg/dL in critically ill patients. This recommendation is based on clinical trials in critically ill patients. In the groups studied, there was no reduction in mortality from intensive treatment targeting near-euglycemic glucose levels compared to conventional management with a target glucose level of <180 mg/dL. There also were reports of harm resulting from intensive glycemic control, including higher rates of severe hypoglycemia and even increased mortality.

Slipped capital femoral epiphysis is most likely in which one of the following patients with no history of trauma? A. A 3-day-old male with a subluxable hip B. A 7-year-old male with groin pain and a limp C. A 13-year-old male with knee pain D. A 16-year-old female with lateral thigh numbness

C. A 13-year-old male with knee pain Slipped capital femoral epiphysis (SCFE) occurs most commonly during the adolescent growth spurt (11-13 years of age for girls, 13-15 years of age for boys). While the cause is unknown, associated factors include anatomic variables such as femoral retroversion or steeper inclination of the proximal femoral physis, in addition to being overweight. African-Americans are affected more commonly as well. The patient may present with pain in the groin or anterior thigh, but also may present with pain referred to the knee. That is also the case for Legg-Calvé-Perthes disease, also known as avascular or aseptic necrosis of the femoral head. This condition most commonly occurs in boys 4-8 years of age. In addition to hip (or knee) pain, limping is a prominent feature. Upper thigh numbness in an adolescent female is a classic symptom of meralgia paresthetica, which is attributed to impingement of the lateral femoral cutaneous nerve in the groin, often associated with obesity or wearing clothing that is too tight in the waist or groin. Developmental dysplasia of the hip is identified by a click during a provocative hip examination of the newborn, using both the Barlow and Ortolani maneuvers to detect subluxation or dislocation.

A 26-year-old pet groomer sustained a dog bite to her left hand 2 hours ago. On examination a 4-cm × 2.5-cm laceration is noted on the thenar eminence of her palm. Although the wound shows some gaping there is minimal active bleeding. No neurovascular injury is noted. Which one of the following is an indication for antibiotics in this patient? A. A wound size >2 cm B. The presence of wound gaping C. A bite involving the hand D. The patient's occupation

C. A bite involving the hand Antibiotic prophylaxis should be used for high-risk bite wounds. Factors associated with a high risk include a bite on an extremity with underlying venous and/or lymphatic compromise, a bite involving the hand, a bite near or in a prosthetic joint, cat bites, crush injuries, delayed presentation, puncture wounds, underlying diabetes mellitus, and immunosuppression. A Cochrane review of nine trials showed no statistical difference in infection rates between prophylaxis and no treatment, except when the bite wound was on the hand. The role of tetanus and rabies prophylaxis should be considered on a case-by-case basis. The other factors listed do not influence whether or not an antibiotic should be prescribed

A 26-year-old male presents with hand pain. He tells you he was out drinking with friends last night and does not remember sustaining any injuries. On examination, there is diffuse swelling and tenderness across the dorsal and medial aspects of the hand. Which one of the following would be the most appropriate treatment? A. A wrist extension splint B. A molded finger splint C. A ulnar gutter splint D. A short arm cast E. Surgical pin fixation

C. A ulnar gutter splint In the radiograph shown, there is a fracture of the fifth metacarpal head, commonly known as a boxer's fracture. There is only slight volar angulation and no displacement. The proper treatment for this fracture is an ulnar gutter splint, which immobilizes the wrist, hand, and fourth and fifth digits in the neutral position. Generally, 3 or 4 weeks of continuous splinting is adequate for healing. Surgical pinning is indicated in cases of significant angulation (35°-40° or more of volar angulation) or in fractures with significant rotational deformity or displacement. The other options listed are not appropriate treatments for this injury. This injury most commonly results from "man-versus-wall" pugilistics, but other mechanisms of injury are possible.

According to the Diagnostic and Statistical Manual of Mental Disorders, 5th edition (DSM-5), the severity of anorexia nervosa is based on which one of the following? A. Refusal to eat B. The frequency of episodes of binge eating or purging behavior C. Body mass index (BMI) D. The presence or absence of amenorrhea E. Orthostatic changes in pulse or blood pressure

C. Body mass index (BMI) According to the DSM-5, the level of severity of anorexia nervosa is based on the patient's body mass index (BMI). Mild is a BMI >17.0 kg/m2, moderate is a BMI of 16.0-16.99 kg/m2, severe is a BMI of 15.0-15.9 kg/m2, and extreme is a BMI <15.0 kg/m2. Recurrent episodes of binge eating or purging behavior help differentiate restricting type from binge-eating/purging type, but do not indicate severity. Orthostatic changes in pulse or blood pressure and refusal to eat are criteria for inpatient hospitalization, but are not part of the classification of severity according to the DSM-5. Amenorrhea can be a clinical sign of anorexia nervosa but is not part of the classification of severity.

You make a diagnosis of depression in a 26-year-old female. Her BMI is 32 kg/m² and she has been trying to lose weight. Which one of the following antidepressants would be LEAST likely to cause her to gain weight? A. Mirtazapine (Remeron) B. Amitriptyline C. Bupropion (Wellbutrin) D. Paroxetine (Paxil) E. Citalopram (Celexa)

C. Bupropion (Wellbutrin) Bupropion is the antidepressant least likely to cause weight gain, and may induce modest weight loss. All of the other choices are more likely to cause weight gain. Among SSRIs, paroxetine is associated with the most weight gain and fluoxetine with the least. Mirtazapine has been associated with more weight gain than the SSRIs.

A 44-year-old male with papulopustular rosacea sees you for follow-up. You have been treating his condition with topical azelaic acid (Finacea), and although his condition is improved he is not satisfied with the results. You suggest adding which one of the following oral medications? A. Clarithromycin (Biaxin) B. Clindamycin (Cleocin) C. Doxycycline D. Erythromycin E. Metronidazole (Flagyl

C. Doxycycline The only FDA-approved oral treatment for acne rosacea is doxycycline at a subantimicrobial dosage (40 mg daily). This does not contribute to antibiotic resistance, even when used over several months, and is better tolerated than higher \dosages. Other antibiotics have limited and low-quality supporting evidence of efficacy and may lead to antibiotic resistance.

A 45-year-old male is seen for a well-demarcated, nonpruritic rash in the right axilla. It is fine-scaled with a cigarette-paper appearance. The rash has a coral-red fluorescence under a Wood's light. Which one of the following is the most likely diagnosis? A. Candidiasis B. Tinea cruris C. Erythrasma D. Inverse psoriasis

C. Erythrasma All of the diagnoses listed are intertriginous rashes but only erythrasma fluoresces with Wood's light. Erythrasma is a superficial gram-positive bacterial infection caused by Corynebacterium minutissimum. The fluorescence is caused by porphyrins. Erythrasma is most often seen between the toe web spaces, followed by the groin and axillae. There are multiple treatments, including topical and oral erythromycins and clindamycins (level of evidence 3, strength of evidence 1)

Which one of the following immunizations is indicated for all pregnant women at any stage of pregnancy? A. MMR B. Varicella C. Influenza D. HPV

C. Influenza Influenza vaccine is indicated for all pregnant women, and there are no known deleterious effects on the course of pregnancy or the fetus. Women are advised to avoid pregnancy for 28 days after receiving MMR or varicella vaccines. HPV vaccine is not recommended during pregnancy.

A 67-year-old male with moderate macrocytosis complains of paresthesias of his feet. If the patient has a borderline low vitamin B12 level, elevated levels of which one of the following would suggest vitamin B12 deficiency? A. Serum gastrin B. Reticulocytes C. Methylmalonic acid D. Serum ferritin E. Serum folate

C. Methylmalonic acid Neurologic symptoms may develop with low-normal vitamin B12 levels in serum. In true vitamin B12 deficiency, methylmalonic acid and homocysteine levels are typically quite elevated, and these return to normal with treatment. Gastrin levels may be abnormal in pernicious anemia, but are not diagnostic alone. High ferritin levels are seen with increased iron stores in the liver, and ferritin levels are used to screen for hemochromatosis.

A 39-year-old male with a history of alcoholism presents to your office with complaints of abdominal pain, vomiting, and nausea following a recent binge. He has eaten little since the onset of his symptoms 3 days ago. Laboratory findings suggest alcoholic ketoacidosis. His serum bicarbonate level is 16.3 mEq/L (N 22.0-26.0). In addition to thiamine, what other treatment should be provided for this patient? A. Bicarbonate and insulin B. Glucagon (GlucaGen) and hydrocortisone C. Normal saline and glucose D. N-acetylcysteine and pyridoxine (vitamin B6)

C. Normal saline and glucose Alcoholic ketoacidosis generally occurs in a patient who has been drinking heavily without eating. Blood glucose levels are usually low or normal, and volume depletion associated with nausea, vomiting, and abdominal pain is the norm. Patients typically have high osmolal and anion gaps. Treatment of alcoholic ketoacidosis includes vigorous volume repletion with normal saline, along with administration of thiamine and glucose. Only in the rare presence of marked acidemia (pH <7.10) is the administration of bicarbonate thought to be necessary. Though insulin levels may be low, hyperglycemia is seldom found. N-acetylcysteine and pyridoxine are not used for the treatment of alcoholic ketoacidosis. Levels of glucagon and hydrocortisone are typically elevated in patients with alcoholic ketoacidosis.

A 70-year-old female presents to your office as a new patient. She is healthy and has no complaints. She walks for exercise 30-45 minutes daily and takes no prescription medications. Her blood pressure is 125/75 mm Hg, heart rate 72 beats/min, and respiratory rate 14/min. On examination she has a systolic crescendo-decrescendo murmur heard loudest at the right upper sternal border. An EKG in the office is within normal limits. Echocardiography shows mild aortic stenosis based on peak aortic jet velocity, aortic valve area, and mean pressure gradient. Her ejection fraction is 55%. At a follow-up visit she states that she continues to be symptom free. Which one of the following should be the next step in the evaluation and management of her aortic stenosis? A. Exercise treadmill testing B. Right heart catheterization C. Repeat echocardiography in 3 years D. Cardiothoracic surgery consultation E. Initiation of statin therapy

C. Repeat echocardiography in 3 years Family physicians see many patients with aortic stenosis (AS) and it is important to know when and if further workup is indicated for asymptomatic patients. Although aortic stenosis can result in adverse cardiac events, most of these events occur in patients who are symptomatic. Thus, the American Heart Association and the American College of Cardiology recommend that asymptomatic patients with mild aortic stenosis undergo repeat echocardiography every 3-5 years. Further workup or treatment is not indicated for patients who have mild AS and are asymptomatic. Exercise treadmill testing may be indicated in patients with severe AS based on echocardiography even if they are asymptomatic. Use of statin drugs has not been shown to slow or stop progression of AS. Right and left heart catheterization can be used in an attempt to resolve discrepancies between symptoms and echocardiographic findings. Because this patient is asymptomatic and her echocardiogram shows only mild AS, left and/or right heart catheterization is not indicated. An ACE inhibitor would be indicated in patients who have a reduced ejection fraction.

A 30-year-old female is referred to you by a local optometrist after she was treated several times for anterior uveitis. You are concerned about an associated systemic disease. She feels well otherwise, and denies back or joint pain, rash, cough, or fever. A chest radiograph reveals enlarged mediastinal lymph nodes. Which one of the following is most likely to be associated with her recurrent uveitis? A. Cat-scratch disease B. Lyme disease C. Sarcoidosis D. Syphilis

C. Sarcoidosis Many patients with uveitis have an associated systemic disease. Some medications may cause secondary uveitis, and conditions such as ocular lymphoma and bloodborne infection may masquerade as primary uveitis. In North America, the most common conditions associated with uveitis are seronegative spondyloarthropathies, sarcoidosis, syphilis, rheumatoid arthritis, and reactive arthritis. All of the conditions listed may be associated with uveitis, but given the chest radiograph findings and clinical scenario in this case, sarcoidosis is most likely.

A 24-year-old gravida 2 para 1 at 9 weeks gestation sees you for a routine prenatal check. She complains of significant nausea, and recommended dietary modifications have not helped. She drives a school bus so she would like to avoid sedating medications. She appears well-hydrated and her examination is otherwise normal. Which one of the following would be best for relieving this patient's nausea? A. Auricular acupressure B. A scopolamine patch (Transderm Scop) C. Vitamin B6 (pyridoxine) D. Methylprednisolone (Medrol)

C. Vitamin B6 (pyridoxine) Nearly 75% of pregnant women are affected by nausea and vomiting of pregnancy. Though dietary modifications are often recommended, there is little evidence to support their use. Vitamin B6 is recommended as first-line therapy. It is safe to use in the first trimester and is associated with less drowsiness compared with other medications. Scopolamine is effective for nausea and vomiting of pregnancy but should be avoided in the first trimester due to the possibility of causing trunk and limb deformities. Likewise, methylprednisolone is also effective but should be avoided in the first trimester as it is associated with an increased risk of cleft palate if used before 10 weeks of gestation. Auricular acupressure has been found to be ineffective.

A mother brings in her 10-year-old son because of a swollen area in his neck that she first noticed yesterday. He has also had symptoms of an upper respiratory infection. On examination the child has a runny nose but otherwise appears well. Palpation reveals a soft, 1.5-cm, slightly tender mass, inferior to the angle of the mandible and anterior to the sternocleidomastoid muscle. The most likely diagnosis is A. thyroglossal duct cyst B. dermoid cyst C. branchial cleft cyst D. thyroid tumor

C. branchial cleft cyst In children, neck masses usually fall into one of three categories: developmental, inflammatory/reactive, or neoplastic. The history and physical examination can help narrow the diagnosis, with location of the mass being particularly helpful. Branchial cleft cysts make up approximately 20% of neck masses in children. They commonly present in late childhood or adulthood, when a previously unrecognized cyst becomes infected. They are most frequently found anterior to the sternocleidomastoid muscle, but can also be preauricular. Thyroglossal duct cysts are located in the midline over the hyoid bone. Frequently, they elevate when the patient swallows. Dermoid cysts are usually mobile, moving with the overlying skin. They can be located in the submental or midline region. Thyroid tumors are also usually located in the midline. Malignant masses are usually hard, irregular, nontender, and fixed.

Examination of a 2-day-old infant reveals flesh-colored papules with an erythematous base located on the face and trunk, containing eosinophils. Which one of the following would be most appropriate at this time? A. An allergy evaluation B. Low-dose antihistamines C. Hydrocortisone cream 0.5% D. A sepsis workup E. Observation only

E. Observation Only This infant has findings consistent with erythema toxicum neonatorum, which usually resolves in the first week or two of life (SOR A). No testing is usually necessary because of the distinct appearance of the lesions. The cause is unknown.

Which one of the following is the recommended duration of dual antiplatelet therapy after placement of a drug-eluting coronary artery stent? A. 1 week B. 1 month C. 2 months D. 3 months E. 1 year

E. 1 year The recommended duration of dual antiplatelet therapy following placement of a drug-eluting coronary artery stent is 1 year (SOR C). The recommended dosages of dual antiplatelet therapy are aspirin, 162-325 mg, and clopidogrel, 75 mg, or prasugrel, 10 mg. Ticlopidine is an option for patients who do not tolerate clopidogrel or prasugrel. The minimum recommended duration of dual antiplatelet therapy is 1 month with bare-metal stents, 3 months with sirolimus-eluting stents, and 6 months with other drug-eluting stents.

Treatment of rhabdomyolysis should routinely include which one of the following? A. Bicarbonate-containing fluids B. Loop diuretics C. Mannitol D. Parenteral corticosteroids E. Isotonic saline

E. Isotonic saline The treatment of rhabdomyolysis includes rapid large infusions of isotonic saline to prevent and treat acute kidney injury, which occurs in 10%-60% of patients. Sodium bicarbonate administration is unnecessary and is not better than normal saline diuresis and increasing urine pH. Loop diuretics and mannitol have little human evidence to support their use. Corticosteroid use is not recommended

A previously healthy 59-year-old male is brought to the emergency department by his wife, who describes symptoms of confusion and ataxia. She also says that he has had a fever and cough for the past 2 weeks. On examination he has a temperature of 39.0°C (102.2°F), a heart rate of 125 beats/min, a respiratory rate of 25/min, a blood pressure of 85/46 mm Hg, and an O2 saturation of 88%. Laboratory findings include a WBC count of 15,500/mm3 (N 4300-10,800), a glomerular filtration rate of 45%, and a hemoglobin level of 9.1 g/dL (N 13.0-18.0). A chest radiograph reveals a large left lower lobe infiltrate. You start the patient on an appropriate antibiotic regimen. Which one of the following is the most appropriate initial treatment of this patient's hypotension? A. Dobutamine B. Dopamine C. Norepinephrine D. Aggressive fluid resuscitation E. Packed red blood cell transfusion

D. Aggressive fluid resuscitation This patient meets the criteria for severe inflammatory response syndrome (SIRS) (fever >38.5°C, heart rate >90 beats/min, respiratory rate >20/min, WBC count >12,000/mm3). He also meets the criteria for severe sepsis, with a positive chest radiograph and evidence of organ hypoperfusion (mental status changes), as well as septic shock (mean arterial pressure <60 mm Hg). The most appropriate initial treatment for patients with hypotension in septic shock is fluid resuscitation (SOR A). While vasopressor therapy is certainly appropriate in septic shock, it should be initiated only after fluid resuscitation fails to restore mean arterial pressure (>65 mm Hg) or when there is evidence of continued organ hypoperfusion. Appropriate antibiotics to cover community-acquired pneumonia are recommended during the first hour of presentation in sepsis (SOR B) but will likely have little effect on acute hypotension. Packed red blood cell transfusion is not indicated in this scenario, as the patient's hemoglobin is above 7 g/dL.

Which one of the following is most appropriate in the management of frostbite? A. Complete surgical debridement of hemorrhagic blisters B. Rapid rewarming with dry heat and warm blankets C. Withholding analgesics to ensure accurate assessment D. Vigorously rubbing affected tissues to restore circulation E. Leaving simple, non-tense areas of clear blistering intact

E. Leaving simple, non-tense areas of clear blistering intact Because of an increased risk of infection, simple non-tense areas of clear blistering in a frostbitten patient are best left intact. Tense or hemorrhagic blisters may be carefully aspirated, but only under sterile conditions. Rapid rewarming of affected areas is best done in a whirlpool bath containing a mild antiseptic at 40°C-41°C. Freezing injuries are extremely painful and analgesics are indicated. Vigorous rubbing of affected tissue is contraindicated, as it may lead to further tissue loss.

A 53-year-old male presents with a 1-day history of swelling in his upper arm. The swelling appeared after a sudden painful "pop" as he was lifting a heavy box. A physical examination reveals a soft to firm, nontender mass in the anterior aspect of the arm, and weakness of forearm supination. Shoulder radiographs are normal. Which one of the following is the most likely diagnosis? A. Acute anterior shoulder dislocation B. Lateral epicondylitis C. Biceps tendinitis D. Biceps tendon rupture

D. Biceps tendon rupture Biceps tendon rupture is one of the most common musculotendinous ruptures. Patients typically present with a visible lump in the upper arm following an audible, painful "pop." The injury typically results from application of an eccentric load to a flexed elbow. Risk factors for biceps tendon rupture include age >40, deconditioning, contralateral biceps tendon rupture, a history of rotator cuff tear, rheumatoid arthritis, and cigarette smoking. Weakness in forearm supination and elbow flexion may be present. The biceps squeeze test and the hook test are both sensitive and specific for diagnosing the condition. Acute anterior shoulder dislocation is typically very painful, with restricted shoulder movements. Lateral epicondylitis results in pain and tenderness over a localized area of the proximal lateral forearm. Biceps tendinitis results in a deep throbbing pain over the anterior shoulder, accompanied by bicipital groove tenderness.

Which one of the following has been shown to be effective for Lyme disease prophylaxis afte removal of an engorged deer tick? A. Amoxicillin B. Ceftriaxone (Rocephin) C. Cefuroxime axetil (Ceftin) D. Doxycycline E. Clarithromycin (Biaxin)

D. Doxycycline While all of the antibiotics listed have been used to treat Lyme disease, the only antibiotic that has been shown to be effective for chemoprophylaxis is doxycycline. A randomized, controlled trial showed that a single 200-mg dose of doxycycline was 87% effective for preventing Lyme disease if given within 72 hours after removal of a deer tick. Nevertheless, a meta-analysis showed that the number needed to treat to prevent one case of erythema migrans was 50, and routine prophylaxis is not recommended. It may be indicated, however, after removal of an engorged nymphal deer tick.

A 32-year-old female comes to your office because she has had increasing difficulty performing her daily tasks for the past 6 months. She says she worries excessively about routine events every day and constantly feels tense, restless, jittery, "on edge," irritable, and unable to relax. She also reports that sometimes her heart pounds and races, her hands feel sweaty and clammy, and her mouth feels like it is "full of cotton." She reports difficulty concentrating and falling asleep at night. A recent TSH level was normal. During your examination you note frequent sighing, a fine tremor in her hands, bitten nails, and clammy hands. Based on her symptoms and examination, which one of the following pharmacologic agents for managing her condition is supported by the best available evidence? A. Alprazolam (Xanax) B. Buspirone C. Hydroxyzine D. Escitalopram (Lexapro) E. Quetiapine (Seroquel)

D. Escitalopram (Lexapro) This patient's symptoms and examination meet the diagnostic criteria for generalized anxiety disorder. Based on the best evidence, pharmacologic agents that are beneficial for treating generalized anxiety disorder in adults include antidepressants such as imipramine, duloxetine, paroxetine, sertraline, escitalopram, and venlafaxine. Buspirone and hydroxyzine are likely to be beneficial but are supported by less convincing evidence and can have unpleasant adverse effects. The antipsychotics and benzodiazepines must be evaluated in terms of trade-offs between benefits and harms. Quetiapine is approved for use in adults as add-on treatment in major depressive disorder for patients who do not have an adequate response to antidepressant therapy alone, for acute depressive episodes in bipolar disorder, for acute manic or mixed episodes in bipolar disorder either alone or with lithium or divalproex, for long-term treatment of bipolar disorder with lithium or divalproex, and for schizophrenia.

A 61-year-old female tells you that her brother was recently diagnosed with hereditary hemochromatosis and his physician suggested that she get tested. She feels well and has no significant health problems. Which one of the following would be most appropriate for initial screening? A. Serum transaminases B. A CBC and a serum iron level C. Testing for the HFE gene D. Ferritin and transferrin saturation E. Total iron binding capacity

D. Ferritin and transferrin saturation The diagnosis of hereditary hemochromatosis requires a random measurement of serum ferritin and calculation of transferrin saturation. The transferrin saturation is calculated by dividing the serum iron level by the total iron binding capacity. If the serum ferritin level is elevated (>200 ng/mL in women) or the transferrin saturation is ³45% the HFE gene should be checked. Measurement of liver transaminases plays a role in determining liver disease but is not helpful in the diagnosis.

A 63-year-old female with community-acquired pneumonia is being treated with appropriate antibiotics. The only abnormality on a basic metabolic panel is a serum sodium level of 121 mEq/L (N 135-145). She reports that her shortness of breath and cough are improving. She has no other complaints on a review of systems. On examination the patient is noted to have normal vital signs and mucous membranes are moist. She has crackles in her right lower lobe. Skin turgor is normal. The remainder of the physical examination is normal. Further testing reveals the following: Urine sodium. . . . . . . . . . . . . . . . . . . . . . . . . . 50 mEq/L Serum osmolality. . . . . . . . . . . . . . . . . . . . . . . 276 mOsm/kg (N 280-285) Urine osmolality. . . . . . . . . . . . . . . . . . . . . . . ..300 mOsm/kg Which one of the following would be most appropriate at this point? A. Intravenous diuretics B. Intravenous hypertonic saline C. Intravenous isotonic saline D. Fluid restriction E. No further interventions

D. Fluid restriction This patient has hypotonic hyponatremia, manifested by low serum osmolality. She is asymptomatic and has no signs of hypovolemia on her laboratory tests or physical examination. Her urine sodium is high and her urine osmolality is low, which indicates the syndrome of inappropriate secretion of antidiuretic hormone (SIADH). This is most likely related to her pneumonia, which is improving. The initial treatment for mild euvolemic hyponatremia is fluid restriction. Intravenous isotonic saline would be indicated for mild hypovolemic hyponatremia. Intravenous hypertonic saline would be indicated for severe hyponatremia with symptoms. Intravenous diuretics would be indicated for hypervolemic hyponatremia, such as in heart failure, along with fluid and sodium restriction.

A 25-year-old male daycare worker presents with a 3-week history of bloating and foul-smelling stools. On examination the patient has mild, diffuse abdominal tenderness and increased bowel sounds. Which one of the following is the most likely cause of this patient's problem? A. Hepatitis A B. Clostridium difficile C. Enterotoxigenic Escherichia coli D. Giardia lamblia E. Campylobacter

D. Giardia lamblia Diarrhea has several causes, requiring different management. In many cases the diarrhea is caused by a viral or bacterial infection that is self-limited and requires only supportive measures. In some cases, however, antibiotic treatment may be needed and it is important to determine the cause of the diarrhea. Patients who have recently been hospitalized for antibiotic treatment are susceptible to infection withClostridium difficile, and should be treated with metronidazole. Travelers to less developed countries oftendevelop travelers' diarrhea from ingesting contaminated food or water. This is most often due toenterotoxigenic Escherichia coli, although travelers can also have Norovirus infections. The most appropriate antibiotic choice in this situation is ciprofloxacin. Patients who become ill after an event where food is served and several attendees have similar symptoms should be suspected of having a Campylobacter infection if the symptoms include bloody diarrhea. This should also be treated with ciprofloxacin. Daycare workers are susceptible to giardiasis, with symptoms including bloating, flatulence, and foul-smelling stools. This can be treated with metronidazole.

A 68-year-old female with diabetes mellitus, coronary artery disease, fibromyalgia, and dyspepsia presents for follow-up. She has been taking omeprazole (Prilosec) for 10 years. It was started during a hospitalization, and her symptoms have returned with previous trials of discontinuation. Which one of the following adverse events is this patient at risk for as a result of her omeprazole use? A. Hypermagnesemia B. Urinary tract infections C. Nephrolithiasis D. Hip fractures

D. Hip fractures Proton pump inhibitors (PPIs), including omeprazole, are generally safe and effective for peptic ulcer disease, gastroesophageal reflux disease, and stress ulcer prevention in critically ill patients. As use has increased, however, risks of long-term use of PPIs have emerged. Currently known risks include increased fractures of the hip, wrist, and spine (SOR B), community-acquired pneumonia (SOR B), Clostridium difficile and other enteric infections (SOR C), hypomagnesemia (SOR B), and cardiac events when coadministered with clopidogrel (SOR B). PPIs may also affect the absorption of vitamins and minerals, including iron, vitamin B12, and folate (SOR C). There is no known association of PPIs with nephrolithiasis or urinary tract infections.

A 56-year-old female comes in for evaluation of gradually worsening right hip pain. She describes her pain as located in the groin and dull in nature, and with activity often notes a clicking sensation associated with sharp pain. On examination her hip range of motion is intact but pain is elicited with extremes of internal and external rotation and her groin pain is exacerbated with the FABER test (knee flexion, abduction and external rotation of the leg until the ankle rests proximal to the contralateral knee) and FADIR test (knee flexion, adduction, and internal rotation of the leg). Which one of the following is the most likely diagnosis? A. Femoral neck fracture B. Femoral hernia C. Trochanteric bursitis D. Hip labral tear

D. Hip labral tear This patient has signs and symptoms of a hip labral tear. This causes dull or sharp groin pain, which in some patients radiates to the lateral hip, anterior thigh, or buttock. The pain usually has an insidious onset, but occasionally begins acutely after a traumatic event. Half of patients also have mechanical symptoms, such as catching or painful clicking with activity. The FADIR and FABER tests are effective for detecting intra-articular pathology (the sensitivity is 75%-96% for the FADIR test and 88% for the FABER test), although neither test has high specificity. Magnetic resonance arthrography is considered the diagnostic test of choice for labral tears, as it has a sensitivity of 90% and an accuracy of 91%. However, if a labral tear is not suspected, less invasive imaging modalities such as plain radiography and conventional MRI should be used first to assess for other causes of hip and groin pain. This patient has no history of trauma or risk factors to suggest a fracture. A femoral hernia would typically present as pain that is worse with straining or lifting, associated with a palpable bulge in the upper thigh. Trochanteric bursitis typically causes lateral hip pain with point tenderness over the greater trochanter of the femur.

A 14-year-old female bumped heads with another player in a soccer game. She was knocked down, appeared briefly dazed, and now has a headache and mild dizziness while seated on the sidelines. Which one of the following would be most appropriate at this point? A. Return to play after symptoms have resolved for at least 30 minutes B. Immediate neuroimaging to rule out intracranial injury C. Complete cognitive and physical rest for 24 hours before returning to normal activities D. Initial complete cognitive and physical rest followed by an individualized graded return to activity E. No sports participation until symptoms have been absent for 1 week

D. Initial complete cognitive and physical rest followed by an individualized graded return to activity This patient has symptoms typical of a mild concussion without loss of consciousness. In such cases standard neuroimaging can be expected to be normal. The evaluation should include a standard concussion assessment tool, and if concussion is suspected the athlete should be removed from play. Complete physical and cognitive rest are required for the first 1-2 days, but return to normal activity must be individualized depending on the course of symptoms and response to gradually increasing activity. Athletes should be completely free of symptoms before returning to sports activities.

Which one of the following is true concerning breast cancer screening? A. It is useful for detecting premalignant conditions B. It can predict which of the discovered cancers are indolent, with a low potential for harm C. The decrease in mortality from breast cancer can be attributed almost entirely to early detection D. It has resulted in an increase in the diagnosis of localized disease E. It has resulted in a significant decrease in the incidence of regional and metastatic disease

D. It has resulted in an increase in the diagnosis of localized disease Breast cancer screening has resulted in an increase in the diagnosis of localized disease without a commensurate decrease in the incidence of more widespread disease. Unfortunately, it cannot predict which of the discovered cancers are more aggressive, and cannot accurately detect premalignant lesions. The decrease in the mortality rate of breast cancer is due both to earlier detection and better follow-up medical care.

Which one of the following is true regarding respiratory syncytial virus (RSV) infection? A. Most infections in the United States occur between August and December B. Corticosteroids should be a routine part of treatment C. The diagnosis is usually based on positive serology D. It is rarely associated with bacterial co-infection

D. It is rarely associated with bacterial co-infection Respiratory syncytial virus (RSV) is a common cause of respiratory tract infections in children. The infections are usually self-limited and are rarely associated with bacterial co-infection, but in very young infants, prematurely born infants, or those with pre-existing heart/lung conditions, the infection can be severe. In North America, RSV season is November to April. Treatment is primarily supportive, including a trial of bronchodilators, with continued use only if there is an immediate response. Corticosteroids and antibiotics are not routinely indicated. Routine laboratory and radiologic studies should not be used in making the diagnosis, as it is based on the history and physical examination

Which one of the following is an appropriate rationale for antibiotic treatment of Bordetella pertussis infections? A. It delays progression from the catarrhal stage to the paroxysmal stage B. It reduces the severity of symptoms C. It reduces the duration of illness D. It reduces the risk of transmission to others E. It reduces the need for hospitalization

D. It reduces the risk of transmission to others Antibiotic treatment for pertussis is effective for eradicating bacterial infection but not for reducing the duration or severity of the disease. The eradication of infection is important for disease control because it reduces infectivity. Antibiotic treatment is thought to be most effective if started early in the course of the illness, characterized as the catarrhal phase. The paroxysmal stage follows the catarrhal phase. The CDC recommends macrolides for primary treatment of pertussis. The preferred antimicrobial regimen is azithromycin for 3-5 days or clarithromycin for 7 days. These regimens are as effective as longer therapy with erythromycin and have fewer side effects. Children under 1 month of age should be treated with azithromycin. There is an association between erythromycin and hypertrophic pyloric stenosis in young infants. Trimethoprim/sulfamethoxazole can be used in patients who are unable to take macrolides or where macrolide resistance may be an issue, but should not be used in children under the age of 2 months. Fluoroquinolones have been shown to reduce pertussis in vitro but have not been shown to be clinically effective

You see a 27-year-old male with autosomal dominant polycystic kidney disease. He has no other medical problems and his renal function has always been normal on annual testing. Today the patient reports his blood pressure at home has been 142-150/84-90 mm Hg. His blood pressure at this visit is 145/88 mm Hg. Which one of the following medications is preferred for the initial management of hypertension in this patient? A. Amlodipine (Norvasc) B. Chlorthalidone C. Furosemide (Lasix) D. Lisinopril (Prinivil, Zestril)

D. Lisinopril (Prinivil, Zestril) Hypertension is the most common manifestation of autosomal dominant polycystic kidney disease and it also contributes to worsening renal function and an increased risk for cardiovascular disease and death. ACE inhibitors such as lisinopril are first-line agents because they have renal protective benefits in addition to their effects on blood pressure. Some studies have suggested they help slow the decline in renal function and help to prevent left ventricular hypertrophy (more so than diuretics or calcium channel blockers). Angiotensin receptor blockers should be reserved for those who cannot tolerate ACE inhibitors.

In which one of the following populations does the U.S. Preventive Services Task Force support ultrasound screening for abdominal aortic aneurysm? A. All men age 55-75 B. Males age 55-75 who currently smoke C. Patients of both sexes age 55-75 who currently smoke D. Men age 65-75 who have ever smoked E. No population group

D. Men age 65-75 who have ever smoked The U.S. Preventive Services Task Force (USPSTF) recommends one-time screening for abdominal aortic aneurysm (AAA) by ultrasonography in men age 65-75 who have ever smoked (Grade B recommendation). The USPSTF recommends against routine screening for AAA in women (Grade D recommendation).

Screening for osteoporosis should be done in which one of the following groups? A. Postmenopausal women B. Women over age 50 with a BMI ≥30 kg/m2 C. Men over age 50 with type 2 diabetes mellitus D. Men over age 70

D. Men over age 70 All women ≥65 (SOR A) and all men ≥70 (SOR C) should be screened for osteoporosis. For men and women age 50-69, the presence of factors associated with low bone density would merit screening. Risk factors include low body weight, previous fracture, a family history of osteoporosis with fracture, a history of falls, physical inactivity, low vitamin D or calcium intake, and the use of certain medications or the presence of certain medical conditions. Chronic systemic diseases that increase risk include COPD, HIV, severe liver disease, renal failure, systemic lupus erythematosus, and rheumatoid arthritis. Endocrine disorders that increase risk include type 1 diabetes mellitus, hyperparathyroidism, hyperthyroidism, Cushing's syndrome, and others. Medications that increase risk include anticonvulsants, corticosteroids, and immunosuppressants. Nutritional risks include celiac disease, vitamin D deficiency, anorexia nervosa, gastric bypass, and increased alcohol or caffeine intake.

A 62-year-old male underwent percutaneous coronary intervention and placement of two stents for a myocardial infarction yesterday. He is currently taking simvastatin (Zocor), aspirin, lisinopril (Prinivil, Zestril), and hydrochlorothiazide. His last LDL-cholesterol level was 70 mg/dL and his blood pressure is 130/80 mm Hg. Which one of the following additions to his current regimen would be most appropriate at this time? A. Amlodipine (Norvasc) B. Diltiazem (Cardizem) C. Verapamil (Calan, Verelan) D. Metoprolol (Lopressor, Toprol-XL) E. No changes

D. Metoprolol (Lopressor, Toprol-XL) β-Blockers are first-line antihypertensive medications for patients with coronary artery disease (CAD) and have been shown to reduce the risk of death by 23% at 2 years. They should also be given to normotensive patients with CAD if tolerated. Cardioselective (β1) β-blockers such as metoprolol and atenolol are preferred, as they cause fewer adverse effects.

A 45-year-old male presents to the emergency department with a complaint of acute, sharp chest pain relieved only by leaning forward. On examination you hear a pericardial friction rub. An EKG shows diffuse ST elevations. Echocardiography reveals a small pericardial effusion. Which one of the following is the most appropriate initial treatment? A. β-Blockers B. Nitrates C. Glucocorticoids D. NSAIDs

D. NSAIDs Patients with acute pericarditis should be treated empirically with colchicine and/or NSAIDs for the first episode of mild to moderate pericarditis. B-Blockers would only be appropriate if the cause of the patient's chest pain were an infarction or ischemia. Nitrates do not relieve the pain of pericarditis. Glucocorticoids are typically reserved for use in patients with severe or refractory cases or in cases where the likely cause of the pericarditis is connective tissue disease, autoreactivity, or uremia (SOR C).

Which one of the following is a significant risk factor for esophageal adenocarcinoma? A. Aspirin therapy B. Ibuprofen therapy C. Helicobacter pylori infection D. Obesity E. Crohn's disease

D. Obesity Esophageal adenocarcinoma has become the predominant type of esophageal cancer in North America and Europe, and gastroesophageal reflux and obesity are the main risk factors. Helicobacter pylori infection, aspirin therapy, NSAID use, and Crohn's disease are not significant risk factors.

A previously healthy 24-year-old female presents with a 10-day history of facial pain and fever. On examination she has tenderness over the maxillary sinus on the left. Which one of the following would be most appropriate for treatment of this patient's condition? A. Intranasal saline flushes B. Intranasal antihistamines C. Oral antihistamines D. Oral antibiotics E. Reassurance only

D. Oral antibiotics While there are several guidelines for the clinical diagnosis of acute bacterial sinusitis (ABS), there is general agreement that patients with a duration of symptoms of at least 10 days without improvement should be treated with antibiotics, including both children and adults (SOR C). Signs and symptoms may include nasal drainage and congestion, facial pressure and/or pain, sinus tenderness, and headache. Recommendations for the duration of treatment vary. One set of guidelines calls for empiric treatment with amoxicillin alone; another recommends going directly to amoxicillin/clavulanate. Suggested alternatives include a "respiratory" quinolone or the combination of a third-generation cephalosporin and clindamycin, particularly in patients with penicillin allergy. Due to the increasing emergence of resistant Streptococcus and Haemophilus species, neither trimethoprim/sulfa¬methoxazole nor macrolides are now recommended for empiric treatment of ABS. Data regarding the efficacy of other measures such as nasal irrigation and the use of decongestants is limited and variable. The most recent guidelines do not recommend the use of decongestants, whether oral or topical.

A 76-year-old female presents with a history of bilateral shoulder pain for the past month. She reports stiffness in the morning for about 1 hour and also reports difficulty getting up when seated in a chair. Acetaminophen is ineffective for her pain. Her erythrocyte sedimentation rate is 65 mm/hr (N 1-25). Which one of the following is the best initial treatment for this condition? A. Oral dexamethasone, 8 mg twice a day B. Oral methylprednisolone (Medrol Dosepak), taper 24 mg to 0 mg over 7 days C. Oral prednisolone (Orapred), 60 mg daily D. Oral prednisone, 15 mg/day E. Intravenous methylprednisolone, 120 mg every 6 hours

D. Oral prednisone, 15 mg/day This patient has polymyalgia rheumatica, based on her history and elevated erythrocytesedimentation rate. The initial treatment is prednisone, 15 mg per day with a slow taper over 1-2 years (SOR C). Alternative treatment includes intramuscular methylprednisolone, 120 mg every 3 weeks

Which one of the following medications is associated with a higher risk of death due to stroke or sudden cardiac death in patients with dementia? A. Diazepam (Valium) B. Fluoxetine (Prozac) C. Paroxetine (Paxil) D. Quetiapine (Seroquel) E. Venlafaxine

D. Quetiapine (Seroquel) In April 2005 the FDA issued a boxed warning for second-generation antipsychotics, including quetiapine, after a meta-analysis demonstrated a 1.6- to 1.7-fold increase in the risk of death associated with their use in elderly patients with dementia, related in part to sudden cardiac death and also to stroke. In June 2008, after two large cohort studies showed a similar risk with first-generation antipsychotics, boxed warnings were added to this class as well. The other medications listed do not have this association or warning.

A copper T 380A intrauterine device (ParaGard) would be preferred over a levonorgestrel-releasing intrauterine device (Mirena) in a patient with a history of which one of the following? A. Nulliparity B. Current smoking C. Acute deep vein thrombosis D. Severe cirrhosis E. Heart failure

D. Severe cirrhosis The intrauterine device (IUD) is a safe and effective method of contraception. There are two main classes of IUDs: the copper T 380A IUD and the levonorgestrel-releasing IUD (14 or 20 :g). There are few contraindications to their use but in certain conditions one class is preferred over the other. Women with severe cirrhosis or liver cancer should not use the levonorgestrel-releasing IUD, and the copper T is preferred. Hormonal contraceptives in general should be avoided in women with severe liver disease, as there is a known association between oral contraceptive use and the growth of hepatocellular adenoma, and this risk is thought to extend to other types of hormonal contraceptives . Breast cancer is another contraindication to use of the levonorgestrel-releasing IUD, and the copper T would be preferred. There is no difference in risk between the copper T and levonorgestrel-releasing IUD with regard to deep vein thrombosis/pulmonary embolism. However, the IUD is preferable to contraceptives containing estrogen. IUDs can be used in nulliparous women and either type may be used, although there is some evidence that there are fewer complications with the levonorgestrel-releasing IUD. Smoking does not preclude the use of either type of IUD. Patients with heart failure may use either type of IUD as well. Women with controlled hypertension may use either form, but there is a slight risk from use of the levonorgestrel-releasing IUD in women with uncontrolled hypertension, although the benefits outweigh the risks.

Which one of the following is an indication for a second dose of pneumococcal polysaccharide vaccine in children? A. A cerebrospinal fluid leak B. Cyanotic congenital heart disease C. Type 1 diabetes mellitus D. Sickle cell disease E. Chronic bronchopulmonary dysplasia

D. Sickle cell disease Patients with chronic illness, diabetes mellitus, cerebrospinal fluid leaks, chronic bronchopulmonary dysplasia, cyanotic congenital heart disease, or cochlear implants should receive one dose of pneumococcal polysaccharide vaccine after 2 years of age, and at least 2 months after the last dose of pneumococcal conjugate vaccine. Revaccination with polysaccharide vaccine is not recommended for these patients. Individuals with sickle cell disease, those with anatomic or functional asplenia, immunocompromised persons with renal failure or leukemia, and HIV-infected persons should receive polysaccharide vaccine on this same schedule and should also be revaccinated at least 3 years after the first dose.

A 57-year-old male presents to the emergency department complaining of dyspnea, cough, and pleuritic chest pain. A chest radiograph shows a large left-sided pleural effusion. Thoracentesis shows a pleural fluid protein to serum protein ratio of 0.7 and a pleural fluid LDH - serum LDH ratio of 0.8. Which one of the following causes of pleural effusion would be most consistent with these findings? A. Cirrhosis B. Heart Failure C. Nephrotic syndrome D. Superior vena cava obstruction

D. Superior vena cava obstruction The protein and lactate dehydrogenase (LDH) levels in pleural fluid can help differentiate between transudative and exudative effusions. Light's criteria (pleural fluid protein to serum protein ratio >0.5, pleural fluid LDH to serum LDH ratio >0.6, and/or pleural LDH >0.67 times the upper limit of normal for serum LDH) are 99.5% sensitive for diagnosing exudative effusions and differentiate exudative from transudative effusions in 93%-96% of cases. Of the listed pleural effusion etiologies, only pulmonary embolism is exudative. The remainder are all transudative.

A 32-year-old male smoker presents with a 4-day history of progressive hoarseness. He is almost unable to speak, and associated symptoms include a cough slightly productive of yellow sputum, as well as tenderness over the ethmoid sinuses. He is afebrile and has normal ear and lung examinations. His oropharynx is slightly red with no exudate, and examination of his nasal passages reveals mucosal congestion. Which one of the following would be the most appropriate treatment? A. Amoxicillin for 10 days B. Omeprazole (Prilosec), 40 mg daily C. Azithromycin (Zithromax) for 5 days D. Symptomatic treatment only

D. Symptomatic treatment only Acute laryngitis most often has a viral etiology and symptomatic treatment is therefore most appropriate. A Cochrane review concluded that antibiotics appear to have no benefit in treating acute laryngitis. Proton pump inhibitors such as omeprazole can be of benefit in treating chronic laryngitis caused by acid reflux, but not for an acute problem such as the one described.

A 67-year-old male presents with a persistent, intermittent cough. He says that his exercise tolerance has decreased, noting that he becomes short of breath more easily while playing tennis. He smoked briefly while in college but has not smoked for over 45 years, and reports no history of known pulmonary disease. You obtain pulmonary function testing in the office to help you diagnose and manage his respiratory symptoms. His FVC and FEV1/FVC are both less than the lower limit of normal as defined by the Third National Health and Nutrition Examination Survey. Repeat testing following administration of a bronchodilator does not correct these values. Which one of the following would be most appropriate at this time? A. A methacholine challenge test B. A mannitol inhalation challenge test C. Exercise pulmonary function testing D. Testing for diffusing capacity of the lung for carbon monoxide (DLCO)

D. Testing for diffusing capacity of the lung for carbon monoxide (DLCO) An FVC that falls below the lower limit of normal (LLN), defined as the fifth percentile of spirometry data obtained from the Third National Health and Nutrition Examination Survey, is consistent with a restrictive pattern of pulmonary function. An FEV1/FVC less than the LLN is consistent with an obstructive defect. A mixed pattern exists when both values are below the LLN, as in this case. The patient should now be referred for full pulmonary function testing, including diffusing capacity of the lungs for carbon monoxide (DLCO). DLCO is a quantitative measure of gas transfer in the lungs. Diseases that decrease blood flow to the lungs or that damage alveoli will lead to less efficient gas exchange and result in a lower DLCO value. Bronchoprovocation (a methacholine challenge, a mannitol inhalation challenge, or exercise testing) should be performed if pulmonary function test results are normal but exercise- or allergen-induced asthma is suspected.

A 48-year-old male sees you for a routine health maintenance examination. His blood pressure is 142/90 mm Hg and you recommend that he return for a repeat blood pressure measurement. Eight weeks later his blood pressure is 138/88 mm Hg. He denies any symptoms on a review of systems. He tells you that on his 40th birthday he abruptly stopped smoking after smoking a pack of cigarettes a day since his early twenties. He is adopted and cannot provide a family history. According to U.S. Preventive Services Task Force guidelines, which one of the following conditions should this patient be screened for now? A. Abdominal aortic aneurysm B. Peripheral arterial disease C. Colon cancer D. Type 2 diabetes mellitus E. Hemochromatosis

D. Type 2 diabetes mellitus U.S. Preventive Services Task Force (USPSTF) guidelines recommend that asymptomatic adults with sustained blood pressure >135/80 mm Hg be screened for type 2 diabetes mellitus using fasting plasma glucose, a 2-hour glucose tolerance test, or hemoglobin A1c measurements (USPSTF B recommendation). Screening for colon cancer with either annual high-sensitivity fecal occult blood testing, sigmoidoscopy every 5 years, or colonoscopy every 10 years is also recommended for adults between the ages of 50 and 75 years (USPSTF A recommendation). Men who have ever smoked (defined as 100 or more cigarettes) should be screened once for abdominal aortic aneurysm (USPSTF B recommendation) between the ages of 65 and 75. Similar screening is recommended in men who have never smoked, but this is a USPSTF grade C recommendation. No recommendation has been made with regard to screening for peripheral vascular disease, and the recommendation on screening for hemochromatosis is listed as inactive on the USPSTF website.

A right-hand-dominant 38-year-old male comes to your office because of right elbow pain. He recently began participating in a highly competitive adult volleyball league, and 2 weeks after he first began playing he developed mild pain in the medial elbow of his right arm. While completing an overhead serve last night he felt an acute worsening of the elbow pain. After the match he noted bruising over his medial elbow. When you examine him you find bruising and pain to palpation around the medial elbow. With his shoulder in 90° of abduction and external rotation you rapidly flex and extend the elbow while maintaining valgus torque on the elbow (the moving valgus stress test). The patient reports pain between 70° and 120° of flexion. This clinical presentation is most consistent with which one of the following causes of elbow pain? A. Medial epicondylitis B. Biceps tendinopathy C. Cubital tunnel syndrome D. Ulnar collateral ligament injury E. Triceps tendinopathy

D. Ulnar collateral ligament injury This patient has injured his ulnar collateral ligament (UCL). The UCL is the primary restraint to valgus stress on the elbow during overhead throwing. These injuries often occur in athletes participating in sports that require overhead throwing, such as baseball, javelin, and volleyball. Patients often report a pop followed by immediate pain and bruising around the medial elbow. The moving valgus stress test has 100% sensitivity and 75% specificity for diagnosing UCL injuries. Medial epicondylitis usually presents with an insidious onset of pain related to a recent increase in occupational or recreational activities. Patients also often report weakened grip strength. The point of maximal tenderness is 5-10 mm distal to and anterior to the medial epicondyle. It is most often a tendinopathy of the flexor carpi radialis and the pronator teres. Biceps tendinopathy usually presents with a history of vague anterior elbow pain and a history of repeated elbow flexion with forearm supination and pronation, such as dumbbell curls. Resisted supination produces pain deep in the antecubital fossa. Cubital tunnel syndrome is a neuropathy of the ulnar nerve caused by compression or traction as it passes through the cubital tunnel of the medial elbow. The onset of pain is more insidious than UCL injury, occurring with repetitive activity, and is usually accompanied by numbness and tingling in the ulnar border of the forearm and hand. If it has existed for some time, the intrinsic hand muscle may become weak. Tendinopathy of the triceps insertion is more common in weight lifters or athletes who repetitively extend their elbows against resistance. Pain occurs at the posterior elbow with resisted extension, and tenderness is located over the triceps insertion.

A 68-year-old white female with a several-year history of well-controlled essential hypertension and a history of acute myocardial infarction 2 years ago is brought to the emergency department complaining of sudden, painless, complete loss of vision in her left eye that began 1 hour ago. Her vital signs are stable, and her blood pressure is 148/90 mm Hg. Her corrected visual acuity is: left—absent, with no light perception; right—20/30. The external eye examination is entirely unremarkable. The most likely diagnosis is: A. acute narrow-angle glaucoma B. optic neuritis C. retinal hemorrhage D. central retinal artery occlusion E. central retinal vein occlusion

D. central retinal artery occlusion The retinal findings shown are consistent with central retinal artery occlusion. The painless, unilateral, sudden loss of vision over a period of seconds may be caused by thrombosis, embolism, or vasculitis. Acute narrow-angle glaucoma is an abrupt, painful, monocular loss of vision often associated with a red eye, which will lead to blindness if not treated. In persons with optic neuritis, funduscopy reveals a blurred disc and no cherry-red spot. Occlusion of the central retinal vein causes unilateral, painless loss of vision, but the retina will show engorged vessels and hemorrhages.

The preferred method for diagnosing psychogenic nonepileptic seizures is: A. inducing seizures by suggestion B. postictal prolactin levels C. EEG monitoring D. video-electroencephalography (vEEG) monitoring E. brain MRI

D. video-electroencephalography (vEEG) monitoring Inpatient video-electroencephalography (vEEG) monitoring is the preferred test for the diagnosis of psychogenic nonepileptic seizures (PNES), and is considered the gold standard (SOR B). Video-EEG monitoring combines extended EEG monitoring with time-locked video acquisition that allows for analysis of clinical and electrographic features during a captured event. Many other types of evidence have been used, including the presence or absence of self-injury and incontinence, the ability to induce seizures by suggestion, psychologic tests, and ambulatory EEG. While useful in some cases, these alternatives have been found to be insufficient for the diagnosis of PNES. Elevated postictal prolactin levels (at least two times the upper limit of normal) have been used to differentiate generalized and complex partial seizures from PNES, but are not reliable (SOR B). While prolactin levels are often elevated after an epileptic seizure, they do not always rise, and the timing of measurement is crucial, making this a less sensitive test than was previously believed. Other serum markers have also been used to help distinguish PNES from epileptic seizures, including creatine phosphokinase, cortisol, WBC counts, lactate dehydrogenase, pCO2 , and neuron-specific enolase. These also are not reliable, as threshold levels for abnormality, sensitivity, and specificity have not been determined. MRI is not reliable because abnormal brain MRIs have been documented in as many as one-third of patients with PNES. In addition, patients with epileptic seizures often have normal brain MRIs.

A mother calls for advice regarding her 2-year-old son. She found an open container of immediate-release diltiazem (Cardizem) on the floor, with some spilled and partially chewed tablets, and estimates that her son opened the container about 90 minutes ago. He does not appear to be in any distress. Which one of the following would you advise her to do? (check one) A. Administer syrup of ipecac at home and observe B. Transport the child to the emergency department for gastric lavage C. Transport the child to the emergency department for administration of activated charcoal D. Transport the child to the emergency department for administration of activated charcoal and a cathartic E. Transport the child to the hospital for admission to the pediatric intensive-care unit for observation

E. More than 9500 cases of calcium channel blocker intoxication were reported to U.S. poison control centers in 2005. Substantial toxicity can occur with one or two tablets, and all children suspected of ingesting a calcium channel blocker should be admitted to a pediatric intensive-care unit for monitoring and management. The use of gastric emptying, cathartics, or adsorptive agents is unlikely to be helpful and should be considered only in patients presenting within 1 hour of ingestion, if then. The American Academy of Pediatrics has advised that syrup of ipecac not be kept in the home because of toxicity and dubious benefit.

Which one of the following is true concerning the use of short-acting inhaled β-agonists for asthma? A. They should be given before any inhaled corticosteroid to facilitate lung delivery B. They are ineffective in patients taking β-blockers C. They are less effective than oral β-agonists D. They are less effective than anticholinergic bronchodilators when given with inhaled corticosteroids E. Their effects begin within 5 minutes and last 4-6 hours

E. The effects of short-acting inhaled β-agonists begin within 5 minutes and last 4-6 hours. In the past, giving inhaled β-agonists just before inhaled corticosteroids was felt to improve the delivery and effectiveness of the corticosteroids. However, this has been proven to be ineffective and is no longer recommended. β-Blockers do diminish the effectiveness of inhaled β-agonists, but this effect is not severe enough to contraindicate using these drugs together. Oral β-agonists are less potent than inhaled forms. Similarly, anticholinergic drugs cause less bronchodilation than inhaled β-agonists and are not recommended as first-line therapy.

A 30-year-old male presents with a 2-week history of swelling of the right posterior elbow. He recalls bumping his elbow against a door, but his pain quickly subsided. He began to notice the swelling over the next 2 days. On examination he has normal range of motion with a boggy, nontender mass over the olecranon. Which one of the following would be most appropriate at this point? A. A posterior splint B. Aspiration C. A corticosteroid injection D. A uric acid level and erythrocyyte rate E. A compression dressing

E. A compression dressing Aseptic olecranon bursitis is often preceded by minor trauma to the elbow followed by a nontender, boggy mass over the olecranon. Septic olecranon bursitis causes not just swelling, but also erythema, warmth, and pain. Half of affected individuals will have a fever. If septic bursitis is suspected, aspiration with bursal fluid analysis should be done and antibiotic therapy should be initiated. Aspiration is not recommended for the initial treatment of aseptic bursitis, as complications such as infection may occur. Management initially is with ice, compression dressings, and avoidance of activities that aggravate the problem. If conservative therapy is unsuccessful the problem can be managed by aspiration followed by compression dressings for 2 weeks. The bursa may be injected with a corticosteroid, but this could cause skin atrophy or infection. Surgical bursectomy can be offered for refractory cases lasting over 3 months.

A 55-year-old female presents with lateral hip pain over the outer thigh. She has no history of injury, although she has just begun a walking program to lose weight. She has increased pain when she lies on that side at night. Her examination is unremarkable except that she is overweight and has tenderness over the greater trochanter. There is no pain with internal and external rotation of the hip. A radiograph reveals minimal osteoarthritic changes. Which one of the following would be most appropriate at this point? A. Serum protein electrophoresis B. A bone scan C. A bone density study D. MRI E. A corticosteroid injection

E. A corticosteroid injection Trochanteric bursitis develops insidiously after repetitive use, and the patient may report morning stiffness and pain when lying on the affected side. Palpation of the greater trochanter elicits tenderness, and occasionally swelling may be noted as well. Early injection with a corticosteroid usually produces a satisfactory response.

Which one of the following, when confirmed with a repeat test, meets the diagnostic criteria for diabetes mellitus? A. A fasting blood glucose level of 120 mg/dL B. A 2-hour value of 180 mg/dL on an oral glucose tolerance test C. A random glucose level of 180 mg/dL in a patient with symptoms of diabetes mellitus D. A positive urine dipstick for glucose E. A hemoglobin A1c of 7.0%

E. A hemoglobin A1c of 7.0% An international expert committee issued a report in 2009 recommending that a hemoglobin A1c level ≥6.5% be used to diagnose diabetes mellitus. Other criteria include a fasting plasma glucose level ≥126 mg/dL, a random glucose leve l≥200 mg/dL in a patient with symptoms of diabetes, or a 2-hour oral glucose tolerance test value ≥200 mg/dL. While a urine dipstick may be used to screen for diabetes, it is not a diagnostic test.

A 50-year-old male presents to your office with a 1-hour history of an intense retro-orbital headache. This started while he was jogging and eased somewhat when he stopped, but has persisted along with some pain in his neck. Other than a blood pressure of 165/100 mm Hg, his examination is unremarkable. Noncontrast CT of the head is also unremarkable. His pain has persisted after 2 hours in the emergency department. Which one of the following would be most appropriate at this time? A. MRI of the head B. Angiography C. Nifedipine (Procardia) sublingually D. Sumatriptan (Imitrex) subcutaneously E. A lumbar puncture

E. A lumbar puncture Early diagnosis of a nontraumatic subarachnoid hemorrhage is paramount for achieving a good outcome when a patient presents with a headache that is unusually severe and feels different than other headaches. Risk factors include smoking, hypertension, heavy alcohol use, and a family history of aneurysm or hemorrhagic stroke. The initial evaluation should consist of noncontrast CT of the head (SOR C). If it is negative or equivocal the next step would be to perform a lumbar puncture to determine whether or not the cerebrospinal fluid is xanthochromic. The absence of xanthochromia rules out subarachnoid hemorrhage.

A 30-year-old female reports that she and her husband have not been able to conceive after trying for 15 months. She takes no medications, has regular menses, and has no history of headaches, pelvic infections, or heat/cold intolerance. Her physical examination is unremarkable. Her husband recently had a normal semen analysis. Which one of the following would be the most appropriate next step? A. Observation for 1 year B. TSH, free T4, and prolactin levels C. Hysterosalpingography D. An estradiol level E. A luteal-phase progesterone level

E. A luteal-phase progesterone level Although infertility issues may be very complex, the primary care physician can initiate an appropriate workup. For women who are having regular menstrual cycles, ovulation is very likely. Ovulation can be confirmed by a progesterone level ³5 ng/mL on day 21 of the cycle. If this is the case, tubal patency should be confirmed with hysterosalpingography or laparoscopy. Obstruction or adhesions would require surgical correction, but if there are none, referral for assisted reproductive technology would be appropriate. Should the progesterone level be <5 ng/mL, anovulation should be investigated with TSH, estradiol, FSH, and prolactin levels. Treatment can be initiated if findings reveal the cause of the problem, but if they are unremarkable it is reasonable to try clomiphene to induce ovulation. If this is unsuccessful, referral would be the next step.

The mother of a 6-year-old male is concerned about his snoring, and she recently observed him stop breathing for a few seconds while he was sleeping. He has also been more sleepy during the day recently. His height and weight are normal. Polysomnography confirms obstructive sleep apnea. Which one of the following would be the most appropriate primary treatment? A. Methylphenidate (Ritalin) B. Lorazepam (Ativan) C. Fluoxetine (Prozac) on a daily basis D. A mouthguard E. Adenotonsillectomy

E. Adenotonsillectomy In children, obstructive sleep apnea (OSA) is most often due to enlarged tonsils and adenoids. OSA onset is usually between 2 and 8 years of age, coinciding with peak tonsil growth. Adenotonsillectomy is the primary treatment for most non-obese children with OSA (SOR B). SSRIs are sometimes effective in treating nightmares because these medications can suppress rapid eye movement sleep. Benzodiazepines are an option for treating sleep terrors. Methylphenidate is a stimulant used to treat attention-deficit/hyperactivity disorder and has no benefit for OSA. The use of a mouthguard at night is recommended for management of temporomandibular joint syndrome to reduce excessive teeth grinding during sleep. It is not a treatment for OSA

Screening for chronic hepatitis B infection is NOT recommended for which one of the following? A. Patients on chronic immunosuppressive therapy B. Patients with end-stage renal disease who are on hemodialysis C. Household contacts of individuals with chronic hep B D. Pregnant women with no risk factors for hepatitis B E. All newborns

E. All newborns Worldwide, hepatitis B is a common cause of liver failure, cirrhosis, and hepatocellular carcinoma. The disease characteristically is asymptomatic before such complications develop. Although routine infant vaccination against hepatitis B has greatly decreased the incidence of this infection in the United States, it remains a significant cause of morbidity and mortality both in the United States and globally. Identifying persons infected with hepatitis B allows vaccination of their household contacts and sexual partners, thereby preventing further transmission. It also allows for medical treatment of infected individuals, including antiviral therapy and monitoring for the development of cirrhosis or hepatocellular carcinoma. The CDC recommends screening for hepatitis B in patients on hemodialysis, household contacts of individuals with chronic hepatitis B, patients on immunosuppressive therapy, and all pregnant women. Other individuals who should be screened include anyone exposed to bodily fluids of infected individuals, such as sexual partners or infants of infected mothers. Behavioral risks such as intravenous drug use are also an indication for screening. Patients from areas where HBsAg prevalence is >2% should also be screened.

Which one of the following screening practices is recommended for the adolescent population by the U.S. Preventive Services Task Force? A. Lipid screening for 3 months B. Scoliosis screening C. Testicular examination D. Papanicolaou tests starting 3 years after first sexual intercourse E. Chlamydia screening in sexually active females

E. Chlamydia screening in sexually active females The U.S. Preventive Services Task Force recommends screening for Chlamydia infection in all sexually active, nonpregnant young women under the age of 25 (grade B recommendation). Papanicolaou testing is recommended starting at 21 years of age. Testicular cancer screening, whether by self-examination or as part of the physical examination, is not recommended. Scoliosis screening for asymptomatic adolescents is also not recommended. There is insufficient evidence to recommend for or against lipid screening.

A 70-year-old male who recently moved to your area sees you for the first time. He has a previous history of myocardial infarction, has a pacemaker, and has hypertension that had been well controlled on hydrochlorothiazide and atenolol (Tenormin) for several years. About 6 months ago his previous physician had to add amlodipine (Norvasc) to his regimen. On examination he has mild arteriolar narrowing in his fundi and there is a systolic bruit just to the right of his umbilicus. He has a log of home blood pressure readings that average 138/88 mm Hg for the past 2 months. His serum creatinine level has gone from 1.2 mg/dL to 1.4 mg/dL (N 0.6-1.2) in the past 2 months. Which one of the following would be most appropriate at this time? A. Referral for stent placement B. Scheduling an arteriogram C. A captopril renal scan D. Adding losartan (Cozaar) to his regimen E. Continued monitoring of serum creatinine

E. Continued monitoring of serum creatinine Renal artery stenosis may be present in as many as 5% of patients with hypertension. It is often seen in those who have coronary artery disease and/or peripheral vascular disease. Hypertension requiring four or five drugs to control, abdominal bruits, and development of hyperkalemia or renal insufficiency after initiating therapy with an ACE inhibitor can all point toward renal artery stenosis as a diagnosis. For patients with renal artery stenosis who have good control, no testing is necessary other than monitoring renal function, particularly if an ACE inhibitor or ARB is part of the regimen. Screening tests recommended by clinical guidelines include duplex ultrasonography, CT angiography, or MR cystography (SOR B). Captopril renography was used in the past but is no longer recommended. In the 1990s uncontrolled studies were done that suggested that either stenting or angioplasty resulted in significant blood pressure reduction and reduced renal failure. However, a clinical trial has shown that stenting did not benefit patients when added to comprehensive multifactorial medical therapy.

A mother brings her 5-year-old daughter to see you because she found a mass in the child's neck. The mass appeared over the past week and was preceded by a sore throat. Her pharyngitis is now resolved but she still has a fever, although it is not as high. The mother is most concerned because the mass developed over a short span of time, and it is warm, red, and tender. When asked, she says that her daughter has had no recent exposure to cats. When you examine the child you note that her temperature is 38.0°C (100.4°F). You also find shotty adenopathy in both anterior cervical lymph node chains, and a 2.5-cm warm, firm, moderately tender lymph node in the right anterior cervical chain. The overlying skin is also erythematous. Which one of the following would be the most appropriate management at this time? A. Ultrasonography of the neck mass B. CT with intravenous contrast of the neck mass C. Ultrasound-guided fine-needle aspiration of the mass D. Immediate referral to a head and neck surgeon E. Empiric antibiotic therapy with observation for 4 weeks

E. Empiric antibiotic therapy with observation for 4 weeks This child has cervical lymphadenitis, characterized by systemic symptoms, unilateral lymphadenopathy, skin erythema, node tenderness, and a node that is 2-3 cm in size. The most common organisms associated with lymphadenitis are Staphylococcus aureus and group A Streptococcus. Empiric antibiotic therapy with observation for 4 weeks is acceptable for children with presumed reactive lymphadenopathy. If symptoms do not resolve, or if the mass increases in size during antibiotic treatment, further evaluation is appropriate. When imaging is indicated, ultrasonography is the preferred initial study for most children with a neck mass. CT with intravenous contrast media is the preferred study for evaluating a malignancy or a suspected retropharyngeal or deep neck abscess that may require surgical drainage. If the initial mass is suspicious for malignancy (>3.0 cm in size, hard, firm, immobile, and accompanied by type B symptoms such as fever, malaise, weight loss, or night sweats) immediate referral to a surgeon for evaluation and possible biopsy is appropriate.

A 45-year-old male presents with shortness of breath and a cough. On pulmonary function testing his FVC is <80% of predicted, his FEV1/FVC is 90% of predicted, and there is no improvement with bronchodilator use. The diffusing capacity of the lung for carbon monoxide (DLCO) is also low. Based on these results, which one of the following is most likely to be the cause of this patient's problem? A. Asthma B. Bronchiectasis C. COPD D. Cystic Fibrosis E. Idiopathic pulmonary fibrosis

E. Idiopathic pulmonary fibrosis Based on the results of pulmonary function testing, this patient has a pure restrictive pattern with a low diffusing capacity for carbon monoxide. Pulmonary fibrosis is compatible with this pattern. A patient with any of the other listed diagnoses would be expected to have an obstructive pattern on testing.

A 45-year-old female presents to your office with a 1-month history of pain and swelling posterior to the medial malleolus. She does not recall any injury, but reports that the pain is worse with weight bearing and with inversion of the foot. Plantar flexion against resistance elicits pain, and the patient is unable to perform a single-leg heel raise. Which one of the following is true regarding this problem? A. The patient most likely has a medial ankle sprain B. NSAIDs will improve the long-term outcome C. Injecting a corticosteroid into the tendon sheath of the involved tendon is recommended D. A lateral heel wedge should be prescribed E. Immobilization in a cast boot for 3 weeks is indicated

E. Immobilization in a cast boot for 3 weeks is indicated The diagnosis of tendinopathy of the posterior tibial tendon is important, in that the tendon's function is to perform plantar flexion of the foot, invert the foot, and stabilize the medial longitudinal arch. An injury can, over time, elongate the midfoot and hindfoot ligaments, causing a painful flatfoot deformity. The patient usually recalls no trauma, although the injury may occur from twisting the foot by stepping in a hole. This is most commonly seen in women over the age of 40. Without proper treatment, progressive degeneration of the tendon can occur, ultimately leading to tendon rupture. Pain and swelling of the tendon is often noted, and is misdiagnosed as a medial ankle sprain. With the patient standing on tiptoe, the heel should deviate in a varus alignment, but this does not occur on the involved side. A single-leg toe raise should reproduce the pain, and if the process has progressed, this maneuver indicates progression of the problem. While treatment with acetaminophen or NSAIDs provides short-term pain relief, neither affects long-term outcome. Corticosteroid injection into the synovial sheath of the posterior tibial tendon is associated with a high rate of tendon rupture and is not recommended. The best initial treatment is immobilization in a cast boot or short leg cast for 2-3 weeks.

A resting ankle-brachial index of 1.50 indicates which one of the following? A. Normal circulation to a lower extremity B. Borderline normal circulation which may not be problematic in an asymptomatic patient C. Mild peripheral artery disease in a lower extremity D. Severe peripheral artery disease in a lower extremity E. Incompressible vessels in a lower extremity

E. Incompressible vessels in a lower extremity An ankle-brachial index (ABI) is considered normal between 1.00 and 1.40, borderline from 0.91 to 0.99, and abnormal if <0.90. The lower the ABI, the more severe peripheral artery disease is likely to be. Values greater than 1.40 indicate incompressible vessels and are not reliable. Incompressible vessels may be found in patients with long-standing diabetes mellitus, or in older persons. A toe-brachial index measurement may be used in persons with incompressible arteries of the more proximal lower extremity.

A 58-year-old male delivery truck driver is diagnosed with type 2 diabetes mellitus and after several months of working on lifestyle modification his hemoglobin A1c is 8.0%. You suggest it is time to start a medication to help control his condition but he is very worried about having a "low sugar reaction" that would prevent him from driving. He is on no other medications at this time. His only other health problem is long-standing controlled hypertension. His BMI is 33.1 kg/m2 and his serum creatinine level is 1.2 mg/dL (N 0.6-1.5). Which one of the following medications would be least likely to cause hypoglycemia in this patient? A. Canagliflozin (Invokana) B. Glimepiride (Amaryl) C. Glipizide (Glucotrol) D. Insulin glargine (Lantus) E. Metformin (Glucophage)

E. Metformin (Glucophage) Metformin is an inexpensive first-line oral agent for type 2 diabetes mellitus. Its mechanism of action is to increase the sensitivity of the liver and peripheral tissues to insulin. This assists the patient with weight loss efforts and, unlike insulin secretagogues, has been proven to reduce mortality with long-term use. When metformin is used as monotherapy it is not associated with episodes of hypoglycemia. For many years there has been a concern that metformin can increase the risk for lactic acidosis. This risk has been assumed to be greater in conditions that can lead to tissue hypoperfusion, such as heart failure or hypovolemia, or with renal impairment. The FDA has historically recommended against the use of metformin for any patient with even mild renal impairment (creatinine >1.4 mg/dL for women and >1.5 mg/dL for men). However, a recent meta-analysis did not find supportive evidence for such restrictions. Newer evidence suggests that the use of metformin is safe even with mild to moderate renal impairment (eGFR >30 mL/min) (SOR A).

A 45-year-old female presents with a rash on the central portion of her face. She states that she has intermittent flushing and intense erythema that feels as if her face is stinging. She has noticed that her symptoms can be worsened by sun exposure, emotional stress, alcohol, or eating spicy foods. She has been in good health and has taken conjugated estrogens (Premarin), 0.625 mg daily, since a hysterectomy for benign reasons. A general examination is normal except for erythema of the cheeks and chin. No pustules or comedone formation is noted around her eyes, but telangiectasias are present. Which one of the following would be appropriate in the management of this problem? A. Increasing her estrogen dosage B. Referral to a rheumatologist C. Low-potency non-fluorinated topical corticosteroids D. Oral prednisone E. Metronidazole gel (MetroGel)

E. Metronidazole gel (MetroGel) Rosacea is a relatively common condition seen most often in women between the ages of 30 and 60. Central facial erythema and telangiectasias are prominent early features that may progress to a chronic infiltrate with papules and sometimes sterile pustules. Facial edema also may occur. Some patients develop rhinophyma due to hypertrophy of the subcutaneous glands of the nose. The usual presenting symptoms are central facial erythema and flushing that many patients find socially embarrassing. Flushing can be triggered by food, environmental, chemical, or emotional triggers. Ocular problems occur in half of patients with rosacea, often in the form of an intermittent inflammatory conjunctivitis with or without blepharitis. Management includes avoidance of precipitating factors and use of sunscreen. Oral metronidazole, doxycycline, or tetracycline also can be used, especially if there are ocular symptoms. These are often ineffective for the flushing, so low-dose clonidine or a nonselective β-blocker may be added. Topical treatments such as metronidazole and benzoyl peroxide may also be effective, particularly for mild cases. Other illnesses to consider include acne, photodermatitis, systemic lupus erythematosus, seborrheic dermatitis, carcinoid syndrome, and mastocytosis.

An obese, hypertensive 53-year-old physician suffers a cardiac arrest while making rounds. He is resuscitated after 15 minutes of CPR, but remains comatose. Which one of the following is associated with the lowest likelihood of neurologic recovery in this situation? A. Duration of CPR >10 minutes B. No pupillary light reflex at 30 minutes C. No corneal reflex at 2 hours D. No motor response to pain at 6 hours E. Myoclonic status epilepticus at 24 hours

E. Myoclonic status epilepticus at 24 hours It is difficult to establish a prognosis in a comatose patient after a cardiac arrest. The duration of CPR is not a factor, and the absence of pupillary and corneal reflexes, as well as motor responses to pain, are not reliable predictors before 72 hours. Myoclonic status epilepticus at 24 hours suggests no possibility of a recovery.

Which one of the following is the most appropriate first-line therapy for primary dysmenorrhea? A. Combined monophasic oral contraceptives B. Combined multiphasic oral contraceptives C. Subdermal etonogestrel (Nexplanon) D. Intramuscular medroxyprogesterone (Depo-Provera) E. NSAIDs

E. NSAIDs The first-line treatment for primary dysmenorrhea should be NSAIDs (SOR A). They should be started at the onset of menses and continued for the first 1-2 days of the menstrual cycle. Combined oral contraceptives may be effective for primary dysmenorrhea, but there is a lack of high-quality randomized, controlled trials demonstrating pain improvement (SOR B). They may be a good choice if the patient also desires contraception. Although combined oral contraceptives and intramuscular and subcutaneous progestin-only contraceptives are effective treatments for dysmenorrhea caused by endometriosis, they are not first-line therapy for primary dysmenorrhea.

A 60-year-old male with a long-standing history of hypertension seeks your advice about pain relief from his osteoarthritis. He has tried acetaminophen and topical capsaicin cream without much benefit. He is concerned about media reports of NSAIDs causing heart problems and is unsure which ones would be safest for him to use. Based on current evidence, which one of the following NSAIDs would you recommend as being LEAST likely to be associated with an increased risk of myocardial infarction? A. Celecoxib (Celebrex) B. Diclofenac (Zorvolex) C. Ibuprofen D. Meloxicam (Mobic) E. Naproxen (Naprosyn)

E. Naproxen (Naprosyn) NSAIDs cause an elevation of blood pressure due to their salt and water retention properties. This effect can also lead to edema and worsen underlying heart failure. In addition, all NSAIDs can have a deleterious effect on kidney function and can worsen underlying chronic kidney disease, in addition to precipitating acute kidney injury. Celecoxib, ibuprofen, meloxicam, and diclofenac are associated with an increased risk of cardiovascular adverse effects and myocardial infarction, compared with placebo. However, naproxen has not been associated with an increased risk of myocardial infarction and is therefore preferred over other NSAIDs in patients with underlying coronary artery disease risk factors

A local dentist contacts you for a prescription for the appropriate antibiotic dosage for one of your patients who has an appointment for dental cleaning to eliminate a significant plaque buildup. The patient is a 55-year-old male who has controlled hypertension and mitral valve prolapse with mitral regurgitation. He is allergic to sulfonamides. Which one of the following would be the most appropriate prophylaxis for this patient? A. Amoxicillin, 2 g orally 1 hour prior to the procedure B. Amoxicillin, 3 g orally 1 hour prior to the procedure and 1.5 g orally 6 hours after the procedure C. Ceftriaxone (Rocephin), 1 g intramuscularly 1 hour prior to the procedure D. Clindamycin (Cleocin), 600 mg orally 1 hour prior to the procedure E. No antibiotic prophylaxis

E. No antibiotic prophylaxis According to the American Heart Association's 2007 guidelines, prophylaxis to prevent bacterial endocarditis associated with dental, gastrointestinal, or genitourinary procedures is now indicated only for high-risk patients with prosthetic valves, a previous history of endocarditis, unrepaired cyanotic congenital heart disease (CHD), or CHD repaired with prosthetic material, and for cardiac transplant recipients who develop valvular disease. Based on a risk-benefit analysis in light of available evidence for and against antibiotic prophylaxis, these recommendations specifically exclude mitral valve prolapse and acquired valvular disease, even if they are associated with mitral regurgitation. The American Dental Association has endorsed this guideline.

A 27-year-old female radiology technician developed an area of redness over the left interscapular region while visiting a friend in Paris last week. The rash has progressed to include the lower back and the patient says it itches. She recalls feeling somewhat tired and achy once she arrived in Paris but attributed this to jet lag. She denies any other systemic symptoms. Your examination reveals no significant findings except for the rash. Of the following, which one is most consistent with this patient's history and examination? A. Guttate psoriasis B. Tinea versicolor C. Radiation dermatitis D. Cutaneous T-cell lymphoma E. Pityriasis rosea

E. Pityriasis rosea This presentation is typical of pityriasis rosea. There was a mild prodrome, thought to be jet lag by this patient, followed by the development of an ovoid salmon-colored, slightly raised herald patch, most commonly seen on the trunk. This was followed by an outbreak of multiple smaller, similar lesions that trend along Langer's lines. In this case, clear evidence of the herald patch remains visible in the left interscapular region, which is helpful in confirming the diagnosis. Guttate psoriasis shares some features with pityriasis rosea in that it can appear suddenly and often follows a triggering incident such as a streptococcal infection, which could be confused with a prodromal phase; however, the absence of a herald patch and the smaller but thicker erythematous lesions differentiate psoriasis from pityriasis rosea. Tinea versicolor often involves the upper trunk and may appear as a lightly erythematous, scaling rash, but the onset is more gradual than in this case. Although this patient may be exposed to low levels of radiation in her job, radiation dermatitis requires doses such as those administered in cancer treatment protocols and would generally be limited to the field of exposure. Cutaneous T-cell lymphoma usually presents as a nonspecific dermatitis, most commonly in men over the age of 50. An infectious etiology for pityriasis rosea is strongly suspected, although none has been identified. There is some evidence that the agent may be human herpesvirus 6. The illness generally resolves within 2 months, leaving no residual signs other than postinflammatory hyperpigmentation.

A 63-year-old female with corticosteroid-dependent COPD has developed pneumonia. Which one of the following pathogens should the antibiotic regimen cover in this patient that would be unlikely in someone with pneumonia and otherwise healthy lungs? A. Streptococcus pneumoniae B. Mycoplasma pneumoniae C. Haemophilus influenzae D. Staphylococcus aureus E. Pseudomonas aeruginosa

E. Pseudomonas aeruginosa All of the pathogens listed can cause pneumonia in any patient. However, in patients with chronic lungdisease who are taking corticosteroids, Pseudomonas is more common than in those with otherwise healthy lungs. The antibiotics chosen empirically should cover this pathogen

A 70-year-old male with hypertension, benign prostatic hyperplasia, depression, and well-controlled diabetes mellitus sees you because of increasing fatigue. His medical history also includes stent placement for coronary artery disease. A physical examination is unremarkable except for decreased peripheral pulses. A CBC, basic metabolic profile, hemoglobin A1c level, free T4 level, and TSH level are all normal, except for a serum sodium level of 125 mEq/L (N 135-145). His serum osmolality is 268 mOsm/kg (N 275-290). His urine sodium level is 50 mEq/L (N <20) and his urine osmolality is 300 mOsm/kg. Which one of the patient's medications is most likely to cause this problem? A. Losartan (Cozaar) B. Tamsulosin (Flomax) C. Metformin (Glucophage) D. Atorvastatin (Lipitor) E. Sertraline (Zoloft)

E. Sertraline In patients who are euvolemic but have hyponatremia, decreased serum osmolality, and elevated urine osmolality, the syndrome of inappropriate secretion of antidiuretic hormone (SIADH) is likely. Other causes to rule out include thyroid disorders, adrenal insufficiency, and diuretic use. Renal function has to be normal as well. Common drugs that cause SIADH include SSRIs (particularly in patients over 65), chlorpropamide, barbiturates, carbamazepine, opioids, tolbutamide, vincristine, diuretics, and NSAIDs. Treatment of the problem consists of discontinuing the offending drug. Temporary fluid restriction may also be required.

Many of the changes that occur as part of aging affect pharmacokinetics. Which one of the following is INCREASED in geriatric patients? A. Drug absorption B. The glomerular filtration rate C. Lean body mass D. The volume of distribution of water-soluble compounds such as digoxin E. The percentage of body fat

E. The percentage of body fat The physiologic changes that accompany aging result in altered pharmacokinetics. In older persons there is a relative increase in body fat and a relative decrease in lean body mass, which causes increased distribution of fat-soluble drugs such as diazepam. This also increases the elimination half-life of such medications. The volume of distribution of water-soluble compounds such as digoxin is decreased in older patients, which means a smaller dose is required to reach a given target plasma concentration. There is also a predictable reduction in glomerular filtration rate and tubular secretion with aging, which causes decreased clearance of medications in the geriatric population. The absorption of drugs changes little with advancing age. All of these changes are important to consider when choosing dosages of medications for the older patient.

A 17-year-old male high school football running back is hit on the lower leg by an opposing player's helmet when the other player dives for a fumble. The running back presents to the emergency department after the game with significant swelling and bruising of the lower leg. Symptoms include exceptionally severe pain that is worse with stretching the calf muscles. There is no weakness of the extremity and sensation is intact. You examine the leg and can palpate pulses. Plain radiographs do not show a fracture. Which one of the following should be ordered next? A. Noninvasive arterial ultrasonography of the leg B. Noninvasive venous ultrasonography of the leg C. CT of the calf region D. MRI of the calf region E. Tissue pressure studies

E. Tissue pressure studies This patient has symptoms and findings consistent with acute compartment syndrome, which is an emergency. The diagnostic test is tissue pressure studies. This condition can occur after a severe injury to the extremity, although it can also develop after a relatively minor injury. Associated problems include fractures, a badly bruised muscle, crush injuries, constricting bandages, and bites with swelling.

Intravenous magnesium is used to correct which one of the following arrhythmias? A. Wenckebach second-degree heart block B. Complete heart block C. Idioventricular rhythm D. Reentrant supraventricular tachycardia E. Ventricular tachycardia of torsades de pointes

E. Ventricular tachycardia of torsades de pointes A well-known use of intravenous magnesium is for correcting the uncommon ventricular tachycardia of torsades de pointes. Results of a meta-analysis suggest that 1.2-10.0 g of intravenous magnesium sulfate also is a safe and effective strategy for the acute management of rapid atrial fibrillation.

A 52-year-old female with morbid obesity is incidentally noted to have mildly elevated AST (SGOT) levels. She does not consume alcohol and denies using recreational drugs. A workup for chronic viral hepatitis and hemochromatosis is negative. Which one of the following is most likely to improve her hepatic condition? A. Pentoxifylline B. Simvastatin (Zocor) C. L-carnitine D. Vitamin E E. Weight loss

E. Weight loss Nonalcoholic fatty liver disease is characterized by the accumulation of fat in hepatocytes. It is associated with insulin resistance, central adiposity, increased BMI, hypertension, and dyslipidemia. An incidentally discovered elevated AST level in the absence of alcohol or drug-induced liver disease strongly suggests the presence of nonalcoholic fatty liver disease. The goal of therapy is to prevent or reverse hepatic injury and fibrosis. Diabetes mellitus, hypertension, dyslipidemia, and other comorbid conditions should be appropriately managed. A healthy diet, weight loss, and exercise are first-line therapeutic measures to reduce insulin resistance in patients with nonalcoholic fatty liver disease. Weight loss has been shown to both normalize AST levels and improve hepatic histology. Vitamin E has been shown to improve AST levels but has no impact on liver histology, and pentoxifylline, simvastatin, and L-carnitine have not been shown to consistently improve either AST levels or liver histology

Complications of hypoparathyroidism include A. somnolence B. low vitamin D C. muscle flaccidity D. hyperkalemia E. refractory heart failure

E. refractory heart failure The classic symptoms of hypoparathyroidism are those of insufficient calcium. Typically these include refractory heart failure, tetany, seizures, altered mental status, and stridor. Refractory heart failure is related to the low calcium interfering with the normal contractility of myocytes. Low vitamin D can cause hypocalcemia but is not caused by it. Patients are not at risk for hyperkalemia if they have hypoparathyroidism. Seizures, not somnolence, and muscle twitching, not flaccidity, are symptoms of low calcium.

A 15-year-old male presents to the emergency department after suffering a lateral dislocation of his patella. Which one of the following would be the best method for reducing this dislocation?

It is usually simple to reduce a lateral patellar dislocation, and these injuries rarely require acute surgical management. The proper technique is to have the patient sit or lie with the leg in a flexed position and then apply gentle medial pressure to the patella until the most lateral edge is over the femoral condyle. The leg should then be gently extended and the knee brought into full extension. This should cause the patella to slip back into place, and the knee should then be immobilized

Which one of the following is a physiologic difference between males and females that can affect the pharmacokinetics of medications with a narrow therapeutic index? A. A consistently higher glomerular filtration rate in women B. The typically higher BMI in women C. Smaller fat stores in women D. Greater gastric acid secretion in women E. Slower gastrointestinal transit times in women

There are key physiologic differences between women and men that can have important implications for drug activity. Gastrointestinal transit times are slower in women than in men, which can diminish the absorption of medications such as metoprolol, theophylline, and verapamil. In addition, women should wait longer after eating before taking medications that should be administered on an empty stomach, such as ampicillin, captopril, levothyroxine, loratadine, and tetracycline. Women also secrete less gastric acid than men, so they may need to drink an acidic beverage to aid in absorption of medications that require an acidic environment, such as ketoconazole. Women usually have lower BMIs than men, and may need smaller loading or bolus dosages of medications to avoid unnecessary adverse reactions. Women typically have higher fat stores than men, so lipophilic drugs such as benzodiazepines and neuromuscular blockers have a longer duration of action. Women also have lower glomerular filtration rates than men, resulting in slower clearance of medications that are eliminated renally, such as digoxin and methotrexate

A 65-year-old male presents to an urgent care center with a foot ulcer. His past medical history is significant for hypertension, COPD, and diabetes mellitus. He has been hospitalized several times in the past year for COPD exacerbations and a hip fracture. He does not have any other current problems. On examination he has a temperature of 37.3°C (99.1°F), a pulse rate of 105 beats/min, a respiratory rate of 16/min, and a blood pressure of 142/83 mm Hg. His examination is unremarkable except for a 2-cm ulcer on the ball of his left foot that has 3 cm of surrounding erythema and some purulent drainage. His CBC is normal except for a WBC count of 14,300/mm3 (N 4300-10,800).

This patient has a severe diabetic foot ulcer. It appears to be infected and there are signs of a systemic inflammatory response. This is an indication for intravenous antibiotics. Piperacillin/tazobactam and vancomycin would be the most appropriate choice of antibiotics because together they cover the most common pathogens in diabetic foot ulcers, as well as MRSA, which is present in 10%-32% of diabetic foot ulcers. This patient has recently been hospitalized and would thus be at high risk for a MRSA infection. Moderate to severe diabetic foot ulcers are often polymicrobial and can include gram-positive cocci, gram-negative bacilli, and anaerobic pathogens.

A 5-year-old white male is brought to your office with a chief complaint of chronic nocturnal limb pain. His mother states that his pain is often severe enough that it awakens him at night and she often gives him ibuprofen to help alleviate his calf pain, but she has never seen him limp or heard him complain of pain during the day. She also has not noticed any grossly swollen joints, fever, rash, or weight change. She is concerned because of a family history of juvenile rheumatoid arthritis in a distant cousin. The physical examination is within normal limits, as are a CBC and an erythrocyte sedimentation rate.

This patient has benign nocturnal limb pains of childhood (previously known as "growing pains"). These crampy pains often occur in the thigh, calf, or shin, occur in up to 35% of children 4-6 years of age, and may continue up to age 19. The pathology of these pains is unknown. The pain is nocturnal, without limping or other signs of inflammatory processes. The erythrocyte sedimentation rate and CBC are normal in this condition but testing is indicated in patients with chronic joint pain to rule out malignancy or infection (SOR C). Rheumatoid factor and ANA have a low predictive value in primary care settings and are not indicated in the pediatric population without evidence of an inflammatory process (SOR C). Plain radiographs are more useful for excluding certain conditions such as cancer than for making a diagnosis of arthritis in children (SOR C). Reassurance of the parents is indicated in this situation, along with instruction on supportive care and over-the counter analgesics as necessary.

A 67-year-old male presents with a 10-day history of bilateral shoulder pain and stiffness accompanied by upper arm tenderness. On examination there is soreness about both shoulders and the patient has great difficulty raising his arms above his shoulders. There is no visual disturbance, and no tenderness over the temporal arteries. C-reactive protein is elevated and the erythrocyte sedimentation rate is 65 mm/hr (N 0-17).

This patient has characteristic features of polymyalgia rheumatica, a disease whose prevalence increases with age in older adults but is almost never seen before age 50. Most people will have accompanying systemic symptoms including fatigue, weight loss, low-grade fever, a decline in appetite, and depression. There are no validated diagnostic criteria available to assist in the diagnosis. The treatment response to 15 mg of prednisone daily is dramatic, often within 24-48 hours, and if this response is not seen, alternative diagnoses must be considered. NSAIDs are not useful in the management of polymyalgia rheumatica and, in fact, are associated with high drug morbidity. Ultrasonography may be useful in making the diagnosis, with typical findings of subdeltoid bursitis and tendon synovitis of the shoulders, but synovitis of the glenohumeral joint is less common.

A 36-year-old female presents with a several-week history of polyuria and intense thirst. She currently takes no medications. On examination her blood pressure and pulse rate are normal, and she is clinically euvolemic. Laboratory tests, including serum electrolyte levels, renal function tests, and plasma glucose, are all normal. A urinalysis is significant only for low specific gravity. Her 24-hour urine output is >5 L with low urine osmolality. The most likely cause of this patient's condition is a deficiency of (check one) A. angiotensin II B. aldosterone C. renin D. insulin E. arginine vasopressin

This patient has diabetes insipidus, which is caused by a deficiency in the secretion or renal action of arginine vasopressin (AVP). AVP, also known as antidiuretic hormone, is produced in the posterior pituitary gland and the route of secretion is generally regulated by the osmolality of body fluid stores, including intravascular volume. Its chief action is the concentration of urine in the distal tubules of the kidney. Both low secretion of AVP from the pituitary and reduced antidiuretic action on the kidney can be primary or secondary, and the causes are numerous. Patients with diabetes insipidus present with profound urinary volume, increased frequency of urination, and thirst. The urine is very dilute, with an osmolality <300 mOsm/L. Further workup will help determine the specific type of diabetes insipidus and its cause, which is necessary for appropriate treatment. Low levels of aldosterone, plasma renin activity, or angiotensin would cause abnormal blood pressure, electrolyte levels, and/or renal function. Insulin deficiency results in diabetes mellitus.

A 12-year-old male uses a short-acting bronchodilator three times per week to control his asthma. Lately he has been waking up about twice a week because of his symptoms. Which one of the following medications would be most appropriate?

This patient has moderate persistent asthma. Although many parents are concerned about corticosteroid use in children with open growth plates, inhaled corticosteroids have not been proven to prematurely close growth plates and are the most effective treatment with the least side effects. Scheduled use of a short-acting bronchodilator has been shown to cause tachyphylaxis, and is not recommended. The same is true for long-acting bronchodilators. Leukotriene use may be beneficial, but compared to those using inhaled corticosteroids, patients using leukotrienes are 65% more likely to have an exacerbation requiring systemic corticosteroids.

A 30-year-old ill-appearing male presents with right hand and arm pain and a rapidly expanding area of redness. On examination he has a temperature of 38.9°C (102.0°F), a pulse rate of 120 beats/min, and a blood pressure of 116/74 mm Hg. He also has erythema from the dorsal hand to the elbow, violaceous bullae on the dorsal hand and wrist, and severe pain with dorsiflexion of the wrist or fingers.

This patient has physical findings consistent with a necrotizing skin and soft-tissue infection, or necrotizing fasciitis. Severe pain and skin changes outside the realm of cellulitis, including bullae and deeper discoloration, are strong indications of necrotizing fasciitis. Antimicrobial therapy is essential but is not sufficient by itself; aggressive surgical debridement within 12 hours reduces the risk of amputation and death.

36-year-old obese female presents to your office with a chief complaint of amenorrhea. On examination you note hirsutism and body acne. She is on no medications and a pregnancy test is negative. Serum testosterone is at the upper limits of normal and TSH is within normal limits. In addition to weight loss and exercise, which one of the following would be the most appropriate initial management?

This patient has polycystic ovary syndrome (PCOS), which is characterized by hyperandrogenism on clinical and laboratory evaluations, polycystic ovaries on pelvic ultrasonography, and ovulatory dysfunction. Hyperandrogenism and either polycystic ovaries or ovulatory dysfunction are necessary to make the diagnosis. The first-line recommendation in obese patients is lifestyle modification, but metformin may improve abnormal menstruation (SOR A). Low-dose combined oral contraceptives are more frequently used to reduce the risk of endometrial cancer in patients with chronic anovulation and the resulting unopposed estrogen secretion. This patient does not have thyroid dysfunction, so levothyroxine is not indicated.

A 7-year-old male is brought to your office with a 2-day history of rash. He developed two itchy spots on his legs yesterday and today he has multiple purple, slightly painful lesions on his legs. A few days ago he was ill with cold-like symptoms, stomach pain, and a fever up to 101.2°F. He complained of leg pain at the time and his left ankle is now swollen. His fever resolved 2 days ago and he now feels fine but limps when he walks. On examination he is afebrile with a normal blood pressure and pulse rate. He is active in the examination room. His physical examination is normal except for purpuric lesions on his legsand buttocks and edema and mild pain of the left ankle. A urinalysis is negative. Which one of the following would be most appropriate in the management of this patient? A. Acetaminophen B. Amlodipine (Norvasc) C. Amoxicillin D. Cyclophosphamide E. Prednisone

This patient meets the clinical criteria for Henoch-Schönlein purpura (HSP), an immune-mediated vasculitis found commonly in children under the age of 10. The clinical triad of purpura, abdominal pain, and arthritis is classic. Almost 95% of children with HSP spontaneously improve, so supportive therapy is the main intervention. Acetaminophen or ibuprofen can be used for the arthritic pain. However, ibuprofen should be avoided in those with abdominal pain or known renal involvement. Prednisone has been found to help in those with renal involvement or other complications of the disease such as significant abdominal pain, scrotal swelling, or severe joint pains (SOR B). However, it is not effective for preventing renal disease or reducing the severity of renal involvement, as was once thought

A 30-year-old male presents to your office because he thinks he may be suffering from alcohol withdrawal. He was dependent on alcohol for at least 10 years and has completed treatment programs twice. He had been abstinent for over a year until he began drinking heavily after his wife filed for divorce 2 weeks ago. A friend found him in a bar last night and has kept him from consuming alcohol for the past 12 hours. The patient is now nauseated, miserable, restless, shaky, and sweating, and says he can feel his heart pounding. He has not had any seizures or episodes of delirium tremens. His temperature is 37.5°C (99.6°F), pulse rate 100 beats/min, and blood pressure 150/92 mm Hg. His palms are moist and he has a mild tremor on arm extension. He is oriented but cannot perform serial additions. A CBC, basic metabolic panel, and urine drug screen are normal. You decide that outpatient treatment would be appropriate. Which one of the following alcohol withdrawal management options is supported by the best evidence?

This patient scores in the moderate range for withdrawal severity and is a candidate for pharmacotherapy, based on the Clinical Institute Withdrawal Assessment for Alcohol Scale, Revised, and the Short Alcohol Withdrawal Scale. He also has no known contraindications to outpatient treatment, such as abnormal laboratory results, absence of a support network, acute illness, high risk for delirium tremens, history of alcohol withdrawal seizure, recent long-term intake of large amounts of alcohol, poorly controlled chronic medical conditions, a serious psychiatric condition, severe withdrawal symptoms, or a positive urine drug screen. Benzodiazepines are the preferred medication for treating alcohol withdrawal (SOR A) and preventing alcohol withdrawal seizures. There is no evidence that indicates that any particular medication is superior, but long-acting benzodiazepines are preferred. Neither fixed nor symptom-triggered dosing of benzodiazepines has been shown to be superior to the other. Although anticonvulsants have less abuse potential than benzodiazepines, they do not prevent seizures or delirium tremens. Clonidine and B-blockers can help reduce adrenergic symptoms but do not prevent alcohol withdrawal seizures. Thiamine or magnesium may be appropriate to address nutritional deficiencies resulting from alcoholism but would not reduce withdrawal symptoms.

Which one of the following is true concerning anterior cruciate ligament (ACL) tears? A. The incidence of ACL tears is higher in males than in females B. ACL tears are not associated with early-onset osteoarthritis C. The majority of ACL tears are caused by physical contact D. Strength training can prevent ACL tears

Three trials have shown that neuromuscular training with plyometrics and strengthening reduces anterior cruciate ligament (ACL) tears. Females have a higher rate of ACL tears than males. Early-onset osteoarthritis occurs in the affected knee in an estimated 50% of patients with ACL tears. The ACL typically pops audibly when it is torn, usually with no physical contact.

A healthy 24-year-old male presents with a sore throat of 2 days' duration. He reports mild congestion and a dry cough. On examination his temperature is 37.2°C (99.0°F). His pharynx is red without exudates, and there are no anterior cervical nodes. His tympanic membranes are normal, and his chest is clear. Which one of the following would be most appropriate at this point? A. Analgesics and supportive care only B. A rapid strep test C. A throat culture and empiric treatment with penicillin D. Azithromycin (Zithromax)

he Centers for Disease Control and Prevention (CDC) assembled a panel of national health experts to develop evidence-based guidelines for evaluating and treating adults with acute respiratory disease. According to these guidelines, the most reliable clinical predictors of streptococcal pharyngitis are the Centor criteria. These include tonsillar exudates, tender anterior cervical lymphadenopathy, absence of cough, and history of fever. The presence of three or four of these criteria has a positive predictive value of 40%-60%, and the absence of three or four of these criteria has a negative predictive value of 80%. Patients with four positive criteria should be treated with antibiotics, those with three positive criteria should be tested and treated if positive, and those with 0-1 positive criteria should be treated with analgesics and supportive care only. This patient has only one of the Centor criteria, and should therefore not be tested or treated with antibiotics.

American Urological Association guidelines define asymptomatic microscopic hematuria as which one of the following in the absence of an obvious benign cause?

≥ 3 RBCs/hpf


संबंधित स्टडी सेट्स

Chapter 15- Duties of Fiduciaries

View Set

Brachial Plexus Injuries, Thoracic Wall/Cavity, Lungs

View Set

Making a Difference Unit Test (76%)

View Set

BIO 210- Human Skeleton (206 bones)

View Set

Ch. 9 Inventory Costing & Capacity Analysis

View Set

Research Methods in Social Psychology

View Set

Correct the sentences and write the sentences with the correct punctuation.

View Set